Космос и астрономия

Ответить в тред Ответить в тред
Check this out!
Тред тупых вопросов №153 Édition Louis-Victor-Pierre-Raymond, 7ème duc de Broglie Аноним 09/08/21 Пнд 13:09:39 6726311
BroglieBig.jpg 18Кб, 300x381
300x381
Figure310600a.jpg 154Кб, 875x485
875x485
Тред вопросов о жизни, Вселенной и всём таком.

Спрашиваем то, за что в других местах выдают путёвку в биореактор. Здесь анонимные учёные мирового уровня критически рассмотрят любые гениальные идеи и нарисованные в Paint схемы.

Предыдущий тут: >>662981 (OP)
https://2ch.pm/spc/res/662981.html

Q: Можно быстрее?
A: Можно упасть в пузырь альбукерке, наса уже почти надула его.

Q: Я начитался охуительных историй про уфологию, че делать, нам жопа?
A: Да, тебе жопа, можешь сгонять в зогач или куда оттуда пошлют.

Q: Что будет с человеком в вакууме без скафандра / если он упадет на черную дыру / попробует ступить на поверхность газового гиганта/солнца?
A: Он умрёт.

Q: Почему бы не привязать ракету к воздушному шару или стартовать с горы?
A: Космос - это не как высоко, а как быстро, большая часть энергии ракеты уходит на разгон вбок.
Подробнее тут https://what-if.xkcd.com/58/ (английский) https://chtoes.li/orbital-speed/ (перевод)
Аноним 09/08/21 Пнд 13:12:33 6726332
Поясни за вторую пикчу.
Аноним 09/08/21 Пнд 13:20:28 6726343
>>672633
де Бройль же. Длина волны де Бройля, квантовая механика, корпускулярно-волновой дуализм. Остальное по пику вроде понятно.

мимо
Аноним 09/08/21 Пнд 13:24:53 6726354
>>672633
Электроны могут занимать только те энергетические уровни, где их волна образует стоячую волну.
Аноним 09/08/21 Пнд 13:27:38 6726365
Если вселенная плоская куда она блять может расширяться? Это же нонсенс.
Аноним 09/08/21 Пнд 13:53:28 6726406
>>672631 (OP)
ты че не уведомил о перекоте
Аноним 09/08/21 Пнд 14:23:09 6726467
>>672640
Забыл, теперь уведомил.

>>672241 →
https://www.nasa.gov/mission_pages/station/research/news/iss-20-years-20-breakthroughs

>>672636
Как куда, во все стороны. Во всех местах вселенной постоянно появляется новое пространство.

>>672607 (OP)
>Далекая галактика, соответственно движется быстрее, а Андромеды- медленнее?
В данном конкретном примере галактика Андромеды вообще движется к нам, но это потому что она очень близко, и тут гравитационное взаимодействие с нами оказывается сильнее эффекта от расширения пространства. А так да, чем дальше от нас галактики, тем они быстрее удаляются.

>Это без учета того что мы можем видеть только то, что происходило с галактиками в прошлом, потому что свет от галактики шел до нас много лет?
>Или ученые высчитали на суперкомпьютерах что галактики и в данный момент времени продолжают разлетаться с той же скоростью, что и в момент времени который мы можем наблюдать сейчас из-за конечной скорости света?
Любая информация, которую мы можем получить о других галактиках - это информация, дошедшая до нас со скоростью света. Когда мы говорим, что галактики удаляются, мы имеем в виду, что мы видим, как они удаляются, по дошедшему до нас от них свету. Вообще, нет смысла считать, что мы видим "далёкое прошлое" тех галактик. Прошлое - это то, что могло повлиять на настоящее. Поскольку никакие взаимодействия не могут распространяться быстрее скорости света, то и на нас могли повлиять только события, произошедшие в тех галактиках до или в момент излучения того света, который мы видим, не позже. Поэтому вполне можно считать, что мы видим галактики в настоящем времени.
Аноним 09/08/21 Пнд 15:10:41 6726518
>>672646
>Поэтому вполне можно считать, что мы видим галактики в настоящем времени.
Но может ли быть такое что сейчас все далекие галактики уже перестали разлетаться и начали сближение, как мы с Андромедой, просто эта информация ещё не дошла до нас? Разве не разумней было бы судить о событиях происходящих во вселенной по нашим ближайшим соседям, а не по далеким галактикам?
Аноним 09/08/21 Пнд 15:44:36 6726579
624e8a0e8d32f98[...].jpg 135Кб, 800x600
800x600
Сап спэйсач. Какие материалы и вещества, из которых состоят небесные тела, можно будет добывать в далеком будущем?
Газ, водород, какой-нибудь грунт, чтоб богатые люди себе клумбы из лунных камней или кусков астеройда делали как на пике.
Должно же изучение космоса когда-нибудь стать прибыльным и хоть чуточку полезным?
Аноним 09/08/21 Пнд 15:48:22 67265910
А как ученые узнали что та или иная планета из чего состоит? Их же там не было, они не брали пробы и не исследовали нихуя. Все что им доступно - наблюдение через телескоп. Ну может излучение замерить тоже можно. Все вроде...
Аноним 09/08/21 Пнд 15:48:40 67266011
>>672646
>Во всех местах вселенной постоянно появляется новое пространство.
Нет, вот я сижу сейчас и стена от меня не отдаляется. Никакого пространства не появляется.
Аноним 09/08/21 Пнд 15:57:25 67266312
>>672659
>А как ученые узнали что та или иная планета из чего состоит? Их же там не было, они не брали пробы и не исследовали нихуя. Все что им доступно - наблюдение через телескоп. Ну может излучение замерить тоже можно. Все вроде...

а) https://ru.wikipedia.org/wiki/Спектроскопия
Ты поразишься сколько всего можно узнать через телескоп.

б) Излучений чертова уйма. Например нейтронный детектор используется что бы воду искать на Марсе
https://rg.ru/2012/03/20/mars-site.html

в) с луны грунт привезли

г) на марсе на роверах стоят анализаторы химического состава грунта. Тоже тащемта через спектроскопию в основном емнип.


Аноним 09/08/21 Пнд 16:11:42 67266613
>>672663
А возможно ли обнаружение абсолютно новых химических веществ, которых нет на земле? Типа вся таблица Менделеева состоит из того, что есть на земле или вся наша вселенная состоит из строго ограниченного кол-ва элементов?
Аноним 09/08/21 Пнд 17:45:57 67267814
>>672657
Так дешевле будет в космосе все производить и спускать на Землю что нужно из готовых изделий.
Аноним 09/08/21 Пнд 17:46:42 67267915
>>672636
>Если вселенная плоская

Я думал, что только Земля плоская.
Аноним 09/08/21 Пнд 17:47:54 67268016
>>672660
современная скорость этого приращения 7-8% линейных величин за лярд лет, не думаю что ты можешь измерить это хоть сидя хоть как-то иначе, на фоне всех физических сил это за гранью погрешности вплоть до масштабов самых больших гравитационно связанных объектов
Аноним 09/08/21 Пнд 17:51:17 67268117
>>672666
Возможно, но маловероятно. Новые элементы могут иметь только больший атомный номер, чем известные, а чем выше номер, тем меньше период полураспада такого элемента. То, что сейчас синтезируют на ускорителях, распадается за ёбаные микросекунды.
Аноним 09/08/21 Пнд 19:29:41 67269718
>Автоматический космический аппарат Solar Orbiter пролетел в понедельник около Венеры. Об этом сообщили в Европейском космическом агентстве (ЕКА), передает ТАСС. Пролет прошел на расстоянии около 8 тыс. км от поверхности планеты.
чем пахнет Венера? Иначе какой смысл так близко подлетать было?
Аноним 09/08/21 Пнд 19:37:03 67269919
>>672657
Промышленное освоение астероидов, гугли
Аноним 09/08/21 Пнд 20:36:31 67271020
>>672636
Нету никакого "куда", расширение Вселенной - явление, которое не имеет никаких аналогов в повседневной жизни. Поэтому попытки рассмотрения этого явления через призму обыденного мышления рождают такие ебанутые вопросы.

Расширение Вселенной - это изменение пространства-времени. Меняется метрика, а именно - масштабный фактор. Расстояние между двумя определенными точками растет со временем, при этом сами точки никуда не движутся. Само пространство между ними "раздувается". Причем делает оно это с ускорением, т.е. темп раздувания постоянно увеличивается.
Аноним 10/08/21 Втр 01:27:00 67275421
>>672681
> Возможно, но маловероятно. Новые элементы могут иметь только больший атомный номер, чем известные, а чем выше номер, тем меньше период полураспада такого элемента. То, что сейчас синтезируют на ускорителях, распадается за ёбаные микросекунды.
Правильно ли я понимаю, что условно добавив атом к элементу можно синтезировать новый? Поэтому они так быстро распадаются.
Аноним 10/08/21 Втр 02:37:18 67276222
Если ниггеры живут на экваторе и они устойчивы к солнечной радиации, почему вместо них в космос летают белые и желтые люди?

Нелогично подвергать себя риску заболеть раком, когда у тебя буквально черномазых биороботов как тараканов за холодильником в питерской коммуналке.
Аноним 10/08/21 Втр 03:09:18 67276423
Какого цвета нейтронные звёзды и ядра звёзд?
Аноним 10/08/21 Втр 05:13:42 67276824
>>672764
Если речь о голых НЗ посреди абсолютного нихуя: это в основном релятивистский объект и в нём дохуя всего происходит, но также они являются и абсолютно чёрными телами, и нехило излучают в спектре АЧТ. Так что в видимом спектре она технически будет казаться примерно белой. Но на практике - очень синей, предельно синей насколько это способен воспринять глаз. Потому что у глаза есть собственный "баланс белого", закодированный в кривой отклика клеток сетчатки. (у камеры точно так же). Излучение АЧТ смещается в верхнюю сторону спектра пропорционально температуре, а НЗ охуительно горячие - поэтому большая часть этого излучения далеко за пределами видимого, где-то в районе рентгена и гаммы. Но глазами тоже будет видно, и сильно смещено в сторону синего.

В реале же чистая НЗ посреди нихуя это редкость - обычно вокруг неё либо крутятся какие-нибудь говна и аккрецируют на неё, либо она крутится в паре с нормальной звездой и перетягивает одеяло на себя, в общем НЗ почти всегда существуют в паре с каким-либо веществом поблизости. Поскольку магнитное поле у нейтронных звезд абсолютно охуевшее, они гнут падающую на них плазму в кривой рог на большой дистанции, так что вокруг них должна быть немаленькая корона из плазмы какого-нибудь цвета, смотря из чего та состоит и какую температуру имеет.
Аноним 10/08/21 Втр 05:19:57 67276925
>>672762
Мамин расист из питерской коммуналки видимо ещё не окончил школу и ему пока не рассказывали, что в черной или загоревшей коже есть меланин, непрозрачный для жесткого ультрафиолета. А не "радиации". Жесткий УФ блокируется даже оконным стеклом.
Аноним 10/08/21 Втр 05:30:32 67277126
>>672769
> Мамин расист из питерской коммуналки видимо ещё не окончил школу и ему пока не рассказывали, что в черной или загоревшей коже есть меланин, непрозрачный для жесткого ультрафиолета. А не "радиации". Жесткий УФ блокируется даже оконным стеклом.
Кому не похуй, если ниггеры одноразовые и их много?
Аноним 10/08/21 Втр 05:50:44 67277227
>>672771
Поридж возвращайся на порашу, это раздел благородных господ.
Аноним 10/08/21 Втр 10:30:01 67278928
>>672710
> расширение Вселенной - явление, которое не имеет никаких аналогов в повседневной жизни
Да не, если вселенная замкнутая и сфероподобная, то аналогией может быть надувание шарика, где вселенная - поверхность этого шарика.
Но вот расширение плоской вселенной понять не могу. Если она вечная и все время, даже во время большого взрыва была вечной, то понять расширение кое-как можно. Но нам же говорят, что вселенная произошла из одной единой точки - сингулярности! И как блять точка может превратится в бесконечную плоскость я понять не могу.
Аноним 10/08/21 Втр 10:47:08 67279529
>>672789
> Но нам же говорят, что вселенная произошла из одной единой точки - сингулярности! И как блять точка может превратится в бесконечную плоскость я понять не могу
никаким "вам" нихуя и не говорили, точка это просто утрированный финал экстраполяции которая по хорошему заканчивается на некотором бесконечно малом объёме и бесконечно большой плотности
Аноним 10/08/21 Втр 14:54:43 67282930
>>672789
У тебя каша в башке. Когда говорят, что Вселенная плоская, имеется в виду, что она обладает евклидовой геометрией на космологических масштабах, т.е. ее кривизна равна нулю. Это совсем не значит, что она бесконечная и уж тем более не значит, что она вечная. Она не вечная и не бесконечная. Ее возраст - 14 млрд лет, а кривизны ее мы не видим, потому что она очень больших размеров - мы видим лишь ее микроскопический кусок, который и выглядит абсолютно плоским (точно так же как находясь на поверхности Земли мы не видим ее кривизны и поверхность для нас выглядит плоской).
>Да не, если вселенная замкнутая и сфероподобная, то аналогией может быть надувание шарика, где вселенная - поверхность этого шарика
Не имеет аналогов в повседневной жизни, т.к. поверхность любого шарика представляет собой двумерную сферу, которую мы наблюдаем из трехмерного пространства. А Вселенная - это трехмерная сфера, которая при этом не погружена ни в какие пространства большей размерности и на которую невозможно взглянуть со стороны.
>Но нам же говорят, что вселенная произошла из одной единой точки
Ни из какой точки Вселенная не произошла, у расширения Вселенной нет центра, оно изотропно, и при экстраполяции этого расширения назад во времени Вселенная не сходится ни в какую точку.
>И как блять точка может превратится в бесконечную плоскость я понять не могу
Никак. Никакой точки и бесконечной плоскости нет.
Аноним 10/08/21 Втр 17:27:21 67285531
Я правильно понимаю что температура звезд как и пламени меняется как светофор при повышении температуры, типа красный - желтый - синий и хз какой еще.

И типа когда когда пламя проходит через весь светофор, то процесс горения выходит на новый уровень и опять все начинается снова, желтый - красный - синий, и так до бесконечности?

Типа пламя ручное на земле, может быть желто-сине-розово-красным. И звезды такого же цвета, хотя их температура очевидно в дохуя раз больше.

И наверное самый тупой вопрос - если для огня требуется кислород, то как в космосе, т.е. где его нет, могут существовать звезды?
Аноним 10/08/21 Втр 18:17:10 67286332
PlanckianLocus.png 460Кб, 1300x1462
1300x1462
>>672855
>Я правильно понимаю что температура звезд как и пламени меняется как светофор при повышении температуры, типа красный - желтый - синий и хз какой еще.
Да. Если конкретнее, то вот пикрелейтед, видишь кривую линию, подписанную Tс(К)? Точки на ней показывают цвет излучения тела, разогретого до соответствующей температуры в Кельвинах.

>И типа когда когда пламя проходит через весь светофор, то процесс горения выходит на новый уровень и опять все начинается снова, желтый - красный - синий, и так до бесконечности?
Нет, с чего ты это вообще взял? Как видишь по графику, линия одна и она сама идёт до бесконечности.

>Типа пламя ручное на земле, может быть желто-сине-розово-красным. И звезды такого же цвета, хотя их температура очевидно в дохуя раз больше.
Температура поверхности Солнца - 5700 К. Любое тело, разогретое на земле до такой температуры, будет светиться таким же цветом. У лампочек накаливания сравнимые температуры.

>если для огня требуется кислород, то как в космосе, т.е. где его нет, могут существовать звезды?
Во-первых, для огня требуется какой-нибудь окислитель, не обязательно кислород. Во-вторых, в звёздах полно разных веществ, в том числе и кислород есть (правда, в очень небольших количествах в сравнении с водородом и гелием). Ну а в-третьих, в звёздах не протекает реакция горения (окисления), а протекает термоядерный синтез, то есть вообще не химический процесс, а ядерный.
Аноним 10/08/21 Втр 19:22:03 67287033
>>672863
>Нет, с чего ты это вообще взял?

Ну я же говорю - сначала пламя на спичке желтое, потом красное на разгоревшимся костре, а при горящем газе уже синие.
Соответственно и звезды бывают желтые, красные, синие, хотя их температура во много раз выше, но цветовой показатель температуры чередуется все так же.

>Ну а в-третьих, в звёздах не протекает реакция горения (окисления), а протекает термоядерный синтез, то есть вообще не химический процесс, а ядерный.

Если бы ты был Василием Ивановичем, а я Петькой, то как бы на пальцах, используя метафоры, сравнения, параллели и т.д. ты бы мне, человеку сельскому и далекому от науки, это объяснил?
Аноним 10/08/21 Втр 19:36:07 67287234
>>672870
Синий цвет пламени газа связан не с его излучением по закону АЧТ, а с изменение энергетических уровней электронов в продуктах горения. Когда электрон переходит с более высокого энергетического уровня на более низкий, излишек энергии он отдаёт в виде фотона. Таким же образом светят газоразрядные лампы, а в них температура вообще считай комнатная.
Аноним 10/08/21 Втр 19:37:58 67287335
>>672870
>Если бы ты был Василием Ивановичем, а я Петькой, то как бы на пальцах, используя метафоры, сравнения, параллели и т.д. ты бы мне, человеку сельскому и далекому от науки, это объяснил?
Серьёзно? Ты не знаешь разницы между химической и ядерной реакцией? А что ты в школе делал, пусть даже и в сельской?
Аноним 10/08/21 Втр 20:01:54 67287536
>>672873
На каком предмете это объясняли?
Химия и физика - с самого начала не разобрался в базовых вещах на первых уроках и в итоге до окончания школы нихуя не понимал.
Математику же, идущую с самого начала. понимал, был круглым отличником до 7 класса, там так же на первых уроках и темах проморгал информацию и пошло как снежный ком, вплоть до сдачи экзаменов от которых решалось останусь я на второй год или нет(из-за геометрии и математики).
Аноним 10/08/21 Втр 20:54:10 67288337
>>672875
И тебе даже не было интересно, чем атомная электростанция отличается от угольной? И почему ядерная бомба настолько недостижимо мощная ёба по сравнению с любой горючей взрывчаткой? Ну и там ладно если бы ты всю жизнь был футбольным подпивасом, но ведь пришёл же на спейсач задавать вопросы про вселенную, то есть любопытство-то есть как бы. Не в претензии, просто редко встречаю таких людей, интересно понять, что они думают.
Аноним 10/08/21 Втр 21:12:47 67289038
1628619162190.jpg 131Кб, 800x811
800x811
>>672883
> редко встречаю таких людей
Аноним 10/08/21 Втр 22:25:53 67291139
>>672883
Я просто слишком ленивый, не люблю читать. Есть конечно нэжйшенел джеографик, но там воду льют очень люто и после просмотра фрагментов где приглашенному ученому ставят задачу объяснить внеочередное явление за полторы минуты, он естественно начинает сокращать свою речь путем всяких сложных неизвестных мне терминов, при объяснении которых потребуются другие термины, их тоже нужно объяснить. И так это может растянуться на несколько часов.

Но никто так кино не делает, хронометраж берегут как и моги зрителей, поэтому он просто говорит абсолютно никому не понятную хуйню, монтажеры подставляют пафосную музыку и 3д мультфильм, где например 3д человек с нелепой анимацией катапультируется с кабины или как камера через модельку юпитера по кинематографичному пролетает. Получается внеочередная научпоп жвачка, которая только пропагандирует изучение науки, но сама в проталкивании знаний в умы даунов вроде меня не участвует.

https://www.youtube.com/watch?v=e3-IGaR5e5o - вообще было бы здорово посмотреть контент про планеты, звезды и т.д. в таком же стиле как этот чел делает про атомную энергетику и оружие. С 3д вставками для ясности и последовательностью.
Аноним 10/08/21 Втр 22:26:48 67291240
Аноним 10/08/21 Втр 22:58:22 67291541
Аноним 10/08/21 Втр 23:19:23 67291942
Аноним 10/08/21 Втр 23:29:40 67292043
>>672919
Ну и в общем, химия - это когда атомы друг с дружкой совокупляются своими электронными оболочками, взаимодействуя электрически, соединяются в молекулы всякие. А ядерные реакции - это когда само ядро расщепляется или наоборот два ядра сталкиваются в одно, один элемент превращается в другой. Ядро очень компактное, при том, что в него упакованы положительные частицы (которые по электрическим законам друг от друга отталкиваются). Уже отсюда должно быть понятно, что силы, которые частицы в ядре удерживают, огромные. Соответственно ядерные реакции - это совсем другой уровень энергий, чем химия.
Аноним 11/08/21 Срд 00:41:40 67293044
Кстати...
Все в мире состоит из атомов, и вода, и земля, отличаются
лишь виды атомов и их строение. Получается что это как строчки кода. Программист пишет один алгоритм из символов - дует ветер, другой - в виртуальном мире появляется смена дня и ночи. Бог Случайность выстраивает атомы так - получается грунт, сяк - вода. Можно ли сказать что мы живем в матрице, где все видимое человеческим глазом - не есть истинное, а лишь текстуры, а на самом деле все вокруг нас это разные алгоритмы и структуры построения разных частиц, строчек кода, разного уровня сложности?

Ну и мои тупорылые вопросы:

Что есть космос, он тоже состоит из каких-либо частиц? Или это воплощение пустоты, которую просто заполняют атомы?

Откуда взялось все материальное, все эти объекты в космическом пространстве, на одном из которых мы живем? Я помню слышал что планеты формировались из какой-то космической пыли, но она то откуда взялась? А откуда взялась эта черная пустота, в которой находится материя, то есть космическое пространство?
Аноним 11/08/21 Срд 01:32:56 67293345
>>672930
>Можно ли сказать что мы живем в матрице, где все видимое человеческим глазом - не есть истинное, а лишь текстуры, а на самом деле все вокруг нас это разные алгоритмы и структуры построения разных частиц, строчек кода, разного уровня сложности?

Можно что угодно сказать, бумага всё стерпит. Если из этого нет каких-то следствий, которые можно экспериментально подтвердить или опровергнуть, всё это не является содержательным высказыванием о реальности.
Аноним 11/08/21 Срд 03:06:45 67294346
>>672930
> Что есть космос
слишком общий вопрос, космос это буквально всё короч
> он тоже состоит из каких-либо частиц? Или это воплощение пустоты, которую просто заполняют атомы?
состоит из самого пространства, по большей части пустого
> Откуда взялось все материальное, все эти объекты в космическом пространстве, на одном из которых мы живем?
если вкратце то изначально от хуйни со стрёмным и вводящим в заблуждение названием "большой взрыв" после первичного расширения и некоторого остывания была сформирована вся первичная материя - гелий с водородом, позже от первых больших звёзд были получены все более тяжелые элементы, которые собрались в пыль и впоследствии сформировали текущие планеты, звёзды и маленькие камушки
Аноним 11/08/21 Срд 05:32:27 67294847
Что будет если кинуть негра в черную дыру размером с шарик для гольфа?
Аноним 11/08/21 Срд 05:33:56 67294948
А если двух негров кинуть, что произойдет?
Аноним 11/08/21 Срд 05:50:44 67295049
Аноним 11/08/21 Срд 05:58:14 67295150
>>672768
>В реале же чистая НЗ посреди нихуя это редкость - обычно вокруг неё либо крутятся какие-нибудь говна и аккрецируют на неё, либо она крутится в паре с нормальной звездой и перетягивает одеяло на себя, в общем НЗ почти всегда существуют в паре с каким-либо веществом поблизости.

Просто мы наблюдаем в основном НЗ рядом с которыми что-то есть.

Большинство нейтронных звезд при рождении получают хороший пинок (pulsar kick) за счет ассиметрии взрыва сверхновой и улетают к чертям собачьим.
Аноним 11/08/21 Срд 07:21:16 67295651
>>672930
>Можно ли сказать что мы живем в матрице, где все видимое человеческим глазом - не есть истинное
Сказать-то можно что угодно, только это нихуя не будет значить.
>Что есть космос, он тоже состоит из каких-либо частиц?
Мы все существуем в четырехмерном пространстве-времени. Космосом и называют это пространство-время, а всякие частицы, поля, излучения - это его содержимое.
>Откуда взялось все материальное, все эти объекты в космическом пространстве, на одном из которых мы живем?
Очень общие вопросы. Объектов вообще во Вселенной много и они все разных типов. На общий вопрос можно дать лишь общий ответ. Сами частицы появились в конце стадии инфляции: энергия вакуума, который обеспечивал инфляцию, перешла в рождение элементарных частиц. Произошло это 14 млрд лет назад, в самом начале Вселенной. Потом из квантовых флуктуаций периода инфляции появились первичные неоднородности плотности вещества, из которых потом выросла крупномасштабная структура, галактики, звезды и мы в конечном итоге.
>Я помню слышал что планеты формировались из какой-то космической пыли
Планеты формируются в протопланетном диске вокруг звезд. Частицы пыли сталкиваются друг с другом, постепенно образуют более крупные структуры - планетезимали, потом эти шарики становятся еще больше - так и появляются планеты.
>но она то откуда взялась?
Откуда взялось вещество и неоднородности в нем я уже написал.
>А откуда взялась эта черная пустота, в которой находится материя, то есть космическое пространство?
Вполне вероятно, из ничего. Тут под "ничто" понимается отсутствие самого пространства. Подобный сценарий делает возможным квантовая механика. В квантовой механике частицы могут туннелировать - преодолевать энергетический барьер, который с точки зрения классической механики преодолеть невозможно. Можно применить этот механизм к пространству как таковому и описать туннелирование целой вселенной из ничего.
Аноним 11/08/21 Срд 07:28:41 67295752
>>672950
А если собрать всех негров и задудосить ЧД размером с мячик для гольфа, хоть один выживет?
Аноним 11/08/21 Срд 13:02:58 67300753
Аноним 11/08/21 Срд 14:34:31 67301554
>>672930
Нет никакого пространства, материи и энергии, есть только поле/струны/браны и тому подобная нех, существующая как минимум в 11 измерениях, меньше наблюдаемая ебань с теорией не сходится. Колебания этой ебани создают иллюзию всего из чего ты стоишь и что тебя окружает на нашем масштабе, включая 3 измерения и время, все пространство, материю, энергию и все взаимодействия.
11/08/21 Срд 18:13:31 67306055
>>673015
Завязывай с наркотиками. Или хотя бы сделай перерыв.
Аноним 11/08/21 Срд 20:32:01 67309256
сверхновая.gif 157Кб, 347x244
347x244
Какая защита потребовалась бы, если находиться в 1000 км от взрыва сверхновой звезды? Алмазный щит? или просто пару метров стали/бетона хватило бы?
Аноним 11/08/21 Срд 20:42:59 67309557
>>673092
В 1000 км ничего не поможет, даже тысячекилометровый слой алмаза. Одного потока нейтрино хватит, чтобы тебя прожарить и испарить, а они спокойно пройдут через любую материю.
Аноним 11/08/21 Срд 20:44:32 67309758
>>673095
То есть, даже в теории нет ничего, что спасло бы на таком расстоянии?
Аноним 11/08/21 Срд 21:17:14 67309859
>>673060
Только ктп, а теперь уже струнная и м-теория имеет шанс уложить всякие кварки, бозоны и прочие бульоны, а они, сюрприз, могут быть только в 11+ измерений, которые для нас сворачиваются в три плюс время. Все старые теории меньших размерностей обосрались и выброшены на помойку. Так что все вокруг нас и мы сами состоим из этой еботы в 11+ измерениях, придется тебе жить с этим.
Аноним 11/08/21 Срд 22:15:34 67310460
>>672631 (OP)
чо такое резонанс шумана простыми словами
Аноним 12/08/21 Чтв 01:28:24 67315061
600px-Schumannr[...].gif 23Кб, 600x400
600x400
>>673104
>чо такое резонанс шумана простыми словами

Бъет молния, выдает уйму радиоволн разной длины, те частоты что затухают хуже всего дают небольшой пик в эфире - это и есть резонанс Шумана.

См картинку - самый острый пик это 50 Гц электросети, Шуман слева, в общем мощность РШ небольшая.

===

Поподробнее:

Бьет разряд молнии. Ее короткий пик выдает радиоволны в уйме частот.

Чем короче частота тем хуже радиоволна огибает поверхность Земли.

Очень длинные волны - 7-20 Гц - это 15-40 т км, сравнимо с размерами Земли, поглощение минимально, добротность порядка 3-8, волна несколько раз обходит Землю.
Аноним 12/08/21 Чтв 02:45:46 67316062
Вот есть типа Земля, на орбите Земли летает Луна. А возможно ли такое, чтобы на орбите Земли находилась мини-Земля размером с Луну, но с атмосферой там и условиями для жизни как на Земле, или этому что-то будет мешать?
Аноним 12/08/21 Чтв 02:46:20 67316163
>>673160
И можно ли будет терраформировать Луну в будущем чтобы создать там условия как на Земле. Луна же меньше и ближе Марса, это должно быть легче
Аноним 12/08/21 Чтв 02:48:46 67316264
>>673161
Магнитное поле слабое в связи с остывшим ядром.
Атмосфера улетучивается.
Нельзя.
Аноним 12/08/21 Чтв 02:59:11 67316365
>>673162
Ну если бы ядро не остыло, можно было бы иметь мини-Землю на орбите Земли?
Аноним 12/08/21 Чтв 08:31:13 67317966
Аноним 12/08/21 Чтв 08:47:36 67318367
>>673162
Но можно купола строить. Или нельзя. Но наверное все же можно. Что не отменяет закапывания глубоко в грунт.
Аноним 12/08/21 Чтв 09:25:33 67318568
Аноним 12/08/21 Чтв 12:04:47 67320269
Ку, спейсач. Хочу разобраться как работает завесное охлаждение жрд. Подскажите где можно с этим разобраться. Добровольского и Кудрявцева не предлагать. Ничо не понял
Аноним 12/08/21 Чтв 12:10:08 67320370
>>673163
Атмосферы нет.

>>673183
микрометиориты и обычные метиориты, через пару лет продырявят твой купол.
Аноним 12/08/21 Чтв 13:00:06 67320771
>>673185
> Атом не делим, я скозал
Верун тут только ты.
Аноним 12/08/21 Чтв 16:03:39 67322472
>>673207
Шизик, завязывай пиздеть с голосами у себя в голове.
Аноним 12/08/21 Чтв 20:54:52 67328373
Зачем появилась вселенная, какой в этом смысл?
Обладает ли вселенная ощущением собственного Я?
Аноним 12/08/21 Чтв 21:55:42 67330174
>>673283
> Зачем появилась вселенная, какой в этом смысл?
ну я захотел, проблеми?
> Обладает ли вселенная ощущением собственного Я?
конечно, передаёт тебе принять таблетки
Аноним 12/08/21 Чтв 22:22:39 67330975
>>673203
Слушай, ты глупый? Я СПРАШИВАЮ ПОХУЙ КАК СЕЙЧАС ВЫГЛЯДИТ И ЧЕМ ЯВЛЯЕТСЯ ЛУНА, МОЖНО ЛИ ЕЁ ЗАМЕНИТЬ ДРУГОЙ ЗЕМЛЕПОДОБНОЙ ПЛАНЕТОЙ РАЗМЕРОМ С ЛУНУ НА ОРБИТЕ ЛУНЫ ВОКРУГ ЗЕМЛИ? СМОЖЕТ ЛИ ТАКОЙ ОБЪЕКТ СУЩЕСТВОВАТЬ ИЛИ ЕМУ ПИЗДЕЦ ПО КАКИМ-ЛИБО ПРИЧИНАМ?
Аноним 12/08/21 Чтв 22:37:35 67331176
>>673162
А если, теоретически, сделать искусственное ядро из тяжёлого метала?
Аноним 13/08/21 Птн 00:26:22 67336277
>>673224
Клоун, отрицающий объективную реальность называет кого-то шизиком, потешно.
Аноним 13/08/21 Птн 00:30:20 67336478
>>673309
> Землеподобная размером с луну
На ноль же делишь, либо землеподобная с водой и атмосферой, либо каменная небольшая безатмосферная дристня, которых дохера даже в нашей системе.
Аноним 13/08/21 Птн 09:24:41 67339779
>>673160
не, сорт оф луна слишком мелкая, вот что-то типа марса, но с магнитным полем было бы правдоподобно
Аноним 13/08/21 Птн 10:14:57 67340580
>>673283
>Зачем появилась вселенная, какой в этом смысл?
Никакого смысла.
>Обладает ли вселенная ощущением собственного Я?
Нет. Вселенная - это четырехмерный пространственно-временной континуум. Какое еще нахуй "ощущение собственного я"? Это же шизотерика какая-то.
Аноним 13/08/21 Птн 10:21:01 67340781
>>673362
>отрицающий объективную реальность
Верунок, плез. Если ты веришь в какие-то маняфантазии, то это еще не значит, что эти маняфантазии являются "объективной реальностью". Они являются лишь частью твоего глупого манямирка. И тот, кто отрицает маняфантазии, составляющие предмет твоей религиозной веры, отрицает не объективную реальность, а, соответственно, твой глупый манямирок. Включи логику, маленький ты дурачок.
Аноним 13/08/21 Птн 10:49:56 67341782
>>673309
Конечно можно. Атмосфера улетучится тысяч за 100 лет, а сильно плотность упадёт через 10к лет. 10к лет - значительный срок! так что можно считать что появись такая планета, люди могли бы там жить бесконечно долго.
Аноним 13/08/21 Птн 10:57:05 67341983
>>673405
Про Больцмановский Мозг слышал?
Аноним 13/08/21 Птн 15:27:52 67349584
>>673283
Зачем появился ты? Какой в этом смысл?

Не "зачем", а "почему".
Аноним 13/08/21 Птн 16:19:17 67350785
>>673283
может наша вселенная это всего лишь одна клетка из триллионов целого организма, а там еще больший пиздец мастштабов
Аноним 13/08/21 Птн 19:35:40 67357086
1628872539157.jpg 784Кб, 2048x1311
2048x1311
Какие есть наработки по НЕФОТОННОЙ передаче информации?

Нейтрино/мюоны/прочее говно.
Видел несколько лет назад статью про эксперимент по передаче сигнала через 100 метров скальной породы с помощью нейтрино. Установка работала, скорость передачи была 100 байт в минуту.

В жизнь не поверю, что военным не нужен скрытый способ передачи информации через непроницаемые для радиоволн среды/в условиях сильных электромагнитных помех.
Это же идеальное дерьмо для связи с подводными лодками, например.

Для связи в условиях радиомолчания и ещё охуилярд применений
Аноним 13/08/21 Птн 19:37:10 67357187
>>673161
ПРОСТО натягиваешь вокруг планеты пленку и заканчиваешь туда воздух.
Аноним 13/08/21 Птн 20:00:07 67358388
>>673570
>скрытый способ передачи информации
С чего ты взял что это будет чем-то скрытнее чем:
>НЕФОТОННОЙ передаче информации?

Если тебе вдруг интересна скрытность, то можно например для обычного радио использовать методы расширения спектра, только с очень большой базой. Вполне можно сделать так чтобы факт передачи нельзя было зафиксировать вовсе (на практике разумеется), разумеется начиная с какого-то определенного расстояния.
Аноним 13/08/21 Птн 20:05:47 67358589
>>673583
Можно не можно, это все лирика. Это перспективная технология, и скрытность там не главная фишка. Я знаю, что можно передавать инфу ниже уровня шума + та же квантовая связь. Особенно если тебе похуй на скорость передачи.

Тут уникальная возможность держать связь с подводными лодками, бункерами и загоризонтными спутниками.
Аноним 13/08/21 Птн 20:14:53 67359390
>>673585
>квантовая связь
Разве оно имеет какое-то отношение к скрытности?
Аноним 13/08/21 Птн 21:56:54 67364291
>>673593
Да, можно передавать инфу на очень низкой мощности
Аноним 14/08/21 Суб 16:59:16 67385392
image.png 31Кб, 781x373
781x373
ААААААААААААААААААААААА
Аноним 14/08/21 Суб 17:15:28 67385793
>>672631 (OP)
Привет всем, посоветуйте пару книг по астрономии, чтобы иметь хоть какие-то знания по этой теме. Спасибо.
Аноним 14/08/21 Суб 17:21:54 67385994
>>673857
>воронцов-вельяминов астрономия 11 класс
>кононович мороз астрономия общий курс астрономии
Аноним 14/08/21 Суб 18:11:56 67386895
>>673853
В чём заключается тупой вопрос?
Аноним 14/08/21 Суб 18:51:42 67388096
>>673868
я пока толковый словарь гуглю, погодь
Аноним 15/08/21 Вск 04:57:19 67396997
Если Солнце в процессе эволюции станет красным гигантом, то разве не нормой должен быть повышающийся температурный тренд на Земле? И наоборот, если температура перестанет повышаться - то это должно быть отклонением от нормы.
Аноним 15/08/21 Вск 07:06:53 67397598
>>673969
Активность солнца растёт, но не такими темпами.
И вообще в тренде обсуждения глобального потепления есть пробел, проблема не в том что становится теплее. Проблема в том что естественные регуляторы температуры и климата разьебываются невиданными темпами.
Аноним 15/08/21 Вск 07:09:28 67397699
>>673975
>естественные регуляторы температуры и климата разьебываются
Это какие?
Аноним 15/08/21 Вск 09:29:19 673988100
>>673969
Повышение светимости Солнца идет на масштабах сотни миллионов лет.
Сейчас Солнце пичот так же и как 500 лет назад. Повышение температуры происходит из-за других факторов.
Аноним 15/08/21 Вск 14:25:29 674052101
Сколько нужно углеродных трубок, чтобы сделать космический лифт? Насколько они будут крепкими?
Не порвутся ли под своим весом+ весом лифта + весом груза? Сколько может выдержать одна углеродная нитка?
Аноним 15/08/21 Вск 21:54:37 674093102
>>674052
пока трубки нужной чистоты не научатся производить в хоть каких-то количествах, на вопросы нет осмысленного ответа, трубки идеальной чистоты должны держать себя, грузы и масштабироваться, те что делают сейчас не держат даже свой вес
Аноним 16/08/21 Пнд 20:06:59 674158103
На сколько от Земли отдаляется Солнце/Луна за век из-за расширения Вселенной?
Аноним 16/08/21 Пнд 20:12:33 674159104
>>674158
Солнечная система и даже галактика и скопление галактик связанна гравитацией, которая автоматом компенсирует космологическое расширение. Более того, все объекты медленно сближаются за счет излучения гравитационных волн.
Аноним 16/08/21 Пнд 21:29:35 674162105
Аноним 16/08/21 Пнд 23:25:26 674170106
>>672948
>в черную дыру размером с шарик для гольфа
Такая ЧД будет весить больше Земли, если я правильно ошибаюсь. Умрет не только негр, но и вообще все вокруг. Если только ты не проводишь этот эксперимент где-нибудь не на Земле.
Аноним 17/08/21 Втр 17:55:19 674258107
>>673162
Разве с увеличением массы атмосферы не повышается вторая космическая для молекул газа её составляющих? По идее, имея огромные запасы газа и плёнки можно было бы сделать плотную атмосферу и на Луне и на Марсе.
Аноним 17/08/21 Втр 18:10:19 674264108
>>674258
нет, с чего ты это взял
Аноним 17/08/21 Втр 18:17:35 674265109
>>674264
Не помню как это называлось, но связывалось с тем, что длина свободного пробега молекулы в плотной атмосфере меньше, из-за чего ей сложнее набрать вторую космическую. Щас я ещё почитаю интернеты, может смогу нагуглить.
Аноним 17/08/21 Втр 18:30:15 674267110
>>674265
Короче не смог найти. Не удивлюсь, если я что-то не так понял или запомнил, но если кто-то понял о чём я говорил и поделится ссылками, то буду благодарен.
Аноним 18/08/21 Срд 11:40:23 674357111
Сколько ракете нужно дельты, чтобы летать на дальность 10 000 км? До какой высоты она при этом должна подниматься?
Аноним 18/08/21 Срд 11:49:14 674358112
>>674357
На пару сот м/с меньше чем на орбиту. По оптимальной траектории надо подниматься на несколько сот км.
Аноним 18/08/21 Срд 15:07:17 674386113
>>674358
Хуёво. А у шаттлов как трансатлантический аборт работал? Какой высоты они бы достигали и какой скорости?
Аноним 18/08/21 Срд 15:33:32 674398114
Чому за большой размер Солнца отвечает давление излучения (ну т.е. только оно упоминается как противодействие гравитации)? А скорость ядер типа вообще ничего не решает? Дохуинная ж температура, должны же что-то делать
Аноним 18/08/21 Срд 16:26:13 674418115
>>674398
Давление излучение растет в четвертой степени от температуры.
Давление невырожденный ядер и электронов почти линейно(там все таки сложная функция, но при высоких температурах растет как в некоторой степени, которая не прерываешь 1,2 ).
С определенной температуры(в районе нескольких сотни тысяч градусов для водородной плазмы) давление изучения превышает давление вещества. А для температуры под миллион давление частиц вещества вносит пренебрежительный малый вклад в суммарное давление.
Ну или под другому можно объяснить так, количество частиц плазмы практически не меняться, а вот количество фотонов в плазме растет с температурой. И при больших температурах фотонный газ остановится основной термодинамической фазой плазмы.

Впрочем все меняется когда наступает квантовое вырождение электронов, при котором давление начинает зависеть от концентрации. Но это не очень актуально для звезд на главной последовательности.
Аноним 18/08/21 Срд 22:16:56 674507116
>>673095
Если они спокойно проходят через любую материю, как они прожарят и испарят тебя?
Аноним 18/08/21 Срд 23:15:02 674516117
>>674418
Эх, ну ладно, спс хоть на этом
Аноним 18/08/21 Срд 23:20:18 674518118
>>674507
Это те что долетают за дохуллионы светолет. Вблизи там такая интенсивность, что даже нейтрино на сдачу тебя захуярят, не говоря об остальном.
Аноним 19/08/21 Чтв 00:11:13 674542119
Коричневые карлики в дипазоне 10,5-12,5 масс Юпитера это полноценные дейтериевые звёзды с продолжительной "дейтериевой главной последовательностью" (до миллиарда лет!)
Вот вам занятный факт.
Аноним 19/08/21 Чтв 09:45:47 674577120
>>674542
Бурые карлики не являются звездами по определению, в их недрах не идут постоянные термоядерные реакции.
Аноним 19/08/21 Чтв 14:24:22 674607121
>>674577
>Бурые карлики
лол маняастроном википедийный, ты?
>не являются звездами по определению
и определение звезды ты конечно доставишь? ну давай, скинь сюда цитатку из википедии
Аноним 19/08/21 Чтв 14:53:53 674608122
>>674607
>пук
Как же быстро порвался этот пердун.
Аноним 19/08/21 Чтв 15:02:50 674609123
>>674608
определение звезды давай, знаток
Аноним 19/08/21 Чтв 15:12:18 674610124
>>674577
>Бурые карлики не являются звездами по определению, в их недрах не идут постоянные термоядерные реакции.
Идут, но только с дейтерием. На D+H давления хватает, на H+H уже нет.
Реакции D+D вроде даже в Юпитере тлеют, но из-за редкости таких событий они крайне медленны и статистически не заметны.

>>673092
>Какая защита потребовалась бы, если находиться в 1000 км от взрыва сверхновой звезды? Алмазный щит? или просто пару метров стали/бетона хватило бы?
В тысяче километров (8% радиуса земли) от поверхности звезды тебя даже спокойное Солнышко испепелит нахуй, какая сверхновая, лол.
Аноним 19/08/21 Чтв 15:24:29 674612125
>>673975
>И вообще в тренде обсуждения глобального потепления есть пробел, проблема не в том что становится теплее. Проблема в том что естественные регуляторы температуры и климата разьебываются невиданными темпами.
Проблема в том, что климатошизики по обе стороны баррикад ошибочно считают Солнце спокойной грелкой. Это ёбаный бурлящий хаос с циклами самой разной длительности (от лет до сотен тысяч лет), которые хуй знает как интерферируют и дают предсказуемость излучения на уровне курса биткоина.
Аноним 19/08/21 Чтв 15:51:35 674615126
Что происходит с фотонами, которые летают во вселенной многие миллиарды лет? Они долетают до края и вылетают из вселенной? Можно ли ловить их гигантским зеркалом и юзать их энергию?
Аноним 19/08/21 Чтв 16:24:21 674621127
>>674615
У Вселенной нет края, пердун. Пиздуй в школу.
Аноним 19/08/21 Чтв 16:52:20 674625128
geosciences-08-[...].png 755Кб, 3517x3382
3517x3382
>>674610
Ещё один теоретик пхпхахааа
Запомни, сынидзе, никакие реакции D(p, γ)3He в Юпитере НЕ идут. Степень выгорания дейтерия резко, очень резко обрывается ниже 10 масс Юпитера. Если для 16MJ выгорание почти 100%, для 11MJ более 70%, то для 10MJ оно меньше 10%, для 9MJ меньше 1%, для 8MJ меньше 0,1% а для 3MJ равно нулю.
>давления не хватает
Давления хватает, не хватает температуры. Если молодой коричневый карлик разогреть в центре до 5 млн. градусов, он вполне себе запалит p-p цикл и стабилизируется на 3 млн. градусах как звезда главной последовательности. То же самое, если разогреть планету в 5 юпитеров до 300 тысяч в центре, планета выйдет на дейтериевую главную последовательность и в силу малой массы будет так экономно расходовать дейтерий, что прогорит несколько сотен миллионов лет не меняя светимости. Это и есть звезда. Дейтериевая. Другое дело что в природе тела с массой 5MJ никогда не разогревтся до поджигания дейтерия. Тем не менее дейтериевая главная посоедовательность существует - для самых лёгких коричневых карликов. Об этом я и говорил выше.
>на H+H не хватает
Хватает. И на Li-H хватает. Тяжёлые коричневые карлики умеют в p-p цикл. Вот только звёздами они не станут, что весьма печально. Слишком велико вырождение в центральной области (никакого "ядра" ни у коричневых карликов ни у лёгких красных кстати нет). На тебе пикчу, образовывайся.
>>674608
Где определение звезды?! Я жду! Или мне называть тебя БЕСПРУФНЫЙ ХУЕСОС?


Аноним 19/08/21 Чтв 16:55:50 674630129
Аноним 19/08/21 Чтв 17:21:20 674635130
>>674630
Беспруфный хуесос ты? Ну ты и чмоня же, ну и лалка обоссаная. Налил говнеца с подливой, тявкнул, а сказать-то тебе и нечего, да? Дааа...
Аноним 19/08/21 Чтв 17:47:06 674639131
Я тут говна всякого начитался, о лунной афере и том сём. И подумол, вот есть Эдик наш Сноуден, с таким то уровнем секретности и тд и тп.
Дак вот, будь пилотируемый полёт на Луну фуфлом, это могло бы стать сильнейшим козырем в его информаторской деятельности, разве не так? к тому же в мемуарах Эдди пишет, шо гуглил это во внутренней сети АНБ
Говном не кидайте, я знаю, вы заняты той же хцйней, но в высокопарных словечках.
Аноним 19/08/21 Чтв 17:50:04 674640132
>>674639
Или про первый полёт на Луну
Или про жизнь одиноких сердец
У каждого фильма - свой конец...

Аноним 19/08/21 Чтв 17:51:58 674641133
>>674635
Пердун, не рвись.
Аноним 19/08/21 Чтв 20:42:31 674671134
>>674639
>я знаю, вы заняты той же хцйней
Ты о чём? Нет, тут доска не про Сноудена и какую-то там информационную деятельность.
Аноним 19/08/21 Чтв 22:16:30 674699135
Если крупная землеподобная планета будет близко обращаться вокруг красного карлана (который каким-то образом не ебошит адовые излучения, что от фонищи обои отславивались), то, ясно дело, год на такой планете будет длиться недолго - от пары недель до нескольких часов. И понятно, что будет приливный захват.
А вот что меня интересует, а может ли быть:
1) у такой планеты магнитное поле?
2) приливный захват быть не полным и планета по чуть-чуть всё равно поворачивает (либо медленно и равномерно, либо рывками сразу проворачиваясь на n градусов зависимости от положении на орбите)?
3) быть жизнь?
Аноним 19/08/21 Чтв 22:48:51 674711136
>>674699
Жизнь разумеется может, никаких ебовых излучений на таких планетах нет - звезда успокаивается за 1 млрд. лет, а потом всё тихо. Красные карлики так-то совсем мало ультрафиолета излучают, они ж холодные. Если всаышка пыханёт всё атмосферой и поглотится. Очки даже не нужны.
Захват очень вероятен, но бывает и резонанс 2:3 например (захват это резонанс 1:1). Тогда вполне себе будет день/ночь, но длинные, по паре недель. Чтобы был резонанс отличный от 1:1 нужно определённое везение и эксцентриситет отличный от 0. С эксцентриситетом может помочь другая планета, соседняя, а может и не помочь.
Так-то это всё отличные планеты, даже в чём-то и лучше, т.к. пока красный карлик молодой он может насветить планете кислородную атмосферу, что гуд.
Магнитное поле тоже будет, у Ганимеда вон есть.
Аноним 19/08/21 Чтв 23:26:07 674719137
>>674507
Изредка даже нейтрино не проходят, иначе бы их не детектировали. Вот от взрыва сверхновой нейтрино настолько сверховердохуя, что даже этого "изредка" хватает с избытком.
Аноним 20/08/21 Птн 00:02:59 674724138
>>674699
>а может ли быть:
>1) у такой планеты магнитное поле?
Хули б нет. Главное, чтоп нутро было расплавленное, и его что-нибудь бурлило. Вон там выше предложили эксцентриситет. Отличная идея: берем спутники Юпитера с резонансом 1:2:4, масштабируем как надо, а Юпитер меняем на красный карлик. Заодно, эксцентриситетом будет раскочегаривать нутро планеты (приливные силы будут нарастать/слабеть по мере приближения/удаления), так что твоя планета не остынет даже за многие миллиарды лет. Такой резонанс, вроде, очень стабилен.
>2) приливный захват быть не полным и планета по чуть-чуть всё равно поворачивает (либо медленно и равномерно, либо рывками сразу проворачиваясь на n градусов зависимости от положении на орбите)?
Может, но сравнительно недолго, пока он еще не установился. Никаких рывков, конечно, просто вращение постепенно замедляется, пока его период не сравняется с периодом обращения вокруг центрального тела. Про 2:3 как у Меркурия выше написали. Это, видимо, редкая штука, но очевидно возможная, даже пример недалеко есть.
>3) быть жизнь?
А хз, скорее всего да. Это все равно гадание на кофейной гуще, на 100% мы знаем только о Земле, ну и на 99% про какой-нибудь Меркурий. Даже про Марс и Венеру есть сомнения насчет прошлого.
Аноним 20/08/21 Птн 05:01:03 674739139
Аноним 20/08/21 Птн 09:06:56 674744140
Поянсите за аномалию поверхности солнца. Почему она горячее (или наоборот холоднее), чем должна быть? Чем это обусловлено? Какие у нас есть обоснования считать это аномалией?
Аноним 20/08/21 Птн 13:04:13 674764141
>>674744
По про какие аномалии говоришь? Про слишком горячую корону?
Так это называют проблемой нагрева солнечной короны. Ее суть в том, что неизвестен точный механизм разгона плазмы и превращение ее в солнечный ветер, средняя скорость которого соответствует плазме в миллионы Кельвин, хотя температура плазмы у поверхности всего несколько тысяч. Тут естественно замещан какой-то нетепловой процесс, который мы прямо не наблюдаем.
Наиболее состоятельная гипотеза состоит в том, что из-за конвекционных процессов у поверхности движение плазмы порождает магнитное поле, выходящее чуть наружу. Конвективные процессы изменчивы во времени, следовательно магнитное поле меняется со временем. Возникает эффект пересоединение магнитных линий. Согласно уравнениям максвелла, изменение магнитного поля порождает электрическое поле. Плазма оказавшиеся в такой области испытывает сильное ускорение и ну или нагрев.
Короче солнечные вспышки на минималках,
Аноним 20/08/21 Птн 16:44:23 674802142
Так блять. Я ток что узнал от Сурдина, что есть проект посадки на Титан, взятие образцов грунта, заправка корабля прям из метановой лужи и прилет на Землю.
Насколько хорошо использовать такие корабли для бомбардировки Марса кометами? Летя с Титана же можно еще пару(?) гравиманевров сделать об гигантов?
Аноним 20/08/21 Птн 17:12:07 674809143
>>674802
>заправка корабля прям из метановой лужи
WUT? Послушать бы что он там бормочет. "Заправка" метаном смысла не имеет без кислорода, а кислорода к метану соотношение 3,5:1. То есть ты чтобы взлететь с Титана 50-тонной ракетой должен будешь посадить 40-тонную ракету, а запустить её от Земли 100500-тонной ракетой.
А 50т это очень мало для Титана, до Земли в таком случае доставишь килограмм 50 (не груза, а 50-кг посадочную капсулу).
Может он имел ввиду электролизить воду ритэгом? Тогда метан тут лишний, и Титан лишний, так можно хоть с Ганимеда запуститься.
Ссылку давай
Аноним 20/08/21 Птн 17:44:39 674824144
Аноним 20/08/21 Птн 18:21:03 674831145
>>674824
На 1:55:00 ничего нет, там космический мусор
Аноним 20/08/21 Птн 22:48:13 674855146
>>674809
>3,5:1
Даже 4 к одному вроде:
CH4 + 2O2 -> СO2 + 2H2O + энергия (12 + 4x1 : 2x2x16 = 1 часть метана на 4 части кислорода)
Но на вывод это не влияет, даже еще хуже все.
Аноним 20/08/21 Птн 22:53:02 674857147
>>674802
>Насколько хорошо использовать такие корабли для бомбардировки Марса кометами?
Нинасколько. Даже если не учитывать пук с "заправкой метаном", ни один гравиманевр кораблем реалистичной в нынешних условиях массы, на комету, считай, никак не повлияет.
Аноним 21/08/21 Суб 00:20:39 674870148
>>674855
Нет не 4, ракетные двигатели работают с избытком горючего. 4:1 ты тоже можешь использовать - стехиометрическое соотношение, и даже не проиграешь в удельном импульсе. Но температура в камере будет немного выше чем на 3,5:1. А удельный импульс тот же самый. А температура выше...
Понял теперь почему с избытком горючего жгут?
Аноним 21/08/21 Суб 00:21:27 674871149
>>674831
Так чего, где там Титан?
Аноним 21/08/21 Суб 08:19:55 674899150
>>674871
Ну до 1.55.30 досмотри будет вопрос про мифы в которые он верил и он про Титан расскажет.
Аноним 21/08/21 Суб 16:09:18 674974151
А если ложный вакуум во вселенной есть, то только во всей сразу или кусками может быть? И если он решит, что пора ебануть в настоящий, то сразу везде сдетонирует или со скоростью света разлетится или вообще остановиться может волна?
Аноним 21/08/21 Суб 21:15:15 675012152
Если звезда вдруг начнёт расширятся и совершать прочие процессы с изменением светимости, излучения, объёма и т.д., как это скажется на орбитах планет?
То есть, вот есть как-Солнце. С ним происходит что-то, например взрыв и последующий сброс вверх во все стороны на 360000 градусов сферы свей короны и т.д., что для наблюдателя на поверхности одной из планет как-Солнца будет выглядеть тупо как разбухание звезды... но что будет с самим пданетами?
По-идее, они же будут по прежнему вращаться вокруг центра масс звезды, а центр масс-то по прежнему будет в её центре. То есть, даже орбита их, по сути, не изменится. Чё им будет?
Ну максимум что я вижу, сброс материи лизнёт дневную поверхность планет (да и то, только близкие, а дальние, вероятно, слишком далеко и их заденет только блеклым рассеянным касанием, тенью настоящего ада ближних планет), ну выжжет к хуям биосферу на дневной стороне, выкипит неглубокие реки и озёра (океаны - вряд ли, ибо сколько же времени подряд сброшенное вещество должно херачить по дну океанов, чтобы получилось полное выжигание и выпаривание?), подпалит и сплавит грунт, песок, камень, да и, наверное, всё.
Получается, планеты легко переживут сброс мантии материнской звездой
Аноним 21/08/21 Суб 21:31:33 675014153
Аноним 21/08/21 Суб 21:48:56 675017154
1.Может ли гравволна достаточной мощности скукоживать тела в чёрные дуры на своём пути? Например, хотя бы нейтронную звезду околокритической массы слиянием больших чд поблизости.
2. Если две чд столкнуть лоб в лоб, волн будет меньше, но гораздо мощнее по энергии? Особенно если изначально задать им околосветовую скорость. Вот тут после слияния она ещё бултыхается, а в рассматриваемой ситуации будет как пружинка дёргаться?
https://www.youtube.com/watch?v=1agm33iEAuo
Аноним 21/08/21 Суб 22:54:25 675025155
>>674974
Детонирующий вакуум - это даже покруче детонирующего Юпитера, лол.
Аноним 21/08/21 Суб 23:03:10 675026156
>>675017
>Может ли гравволна достаточной мощности скукоживать тела в чёрные дуры на своём пути?
Тащемта даже две гравитационных волны достаточной силы при движении навстречу теоретически могут образовать ЧД. Но условия нужны специфические, которых в природе возникнуть не может. И не только гравитационных, например https://en.wikipedia.org/wiki/Kugelblitz_(astrophysics)

В общем смысле любая достаточно высокая концентрация энергии, массы, или их комбинация, превратится в ЧД.
Аноним 21/08/21 Суб 23:06:27 675027157
Аноним 22/08/21 Вск 00:51:34 675046158
>>675026
А если через чд пройдут мощные волны (от других источников), она будет колбаситься, меняться в размерах во время этого? И когда длина волн будет равна/кратна диаметру гс, может, чд в какой-то резонанс войдёт, заиграет гармошкой поляризованного вакуума? Будет ли усиливаться испарение Хокинга?
Аноним 22/08/21 Вск 00:54:53 675047159
>>675046
Хокинг гармошкой пойдёт, будет колбаситься, испарение его усилится. А чёрной дыре всё равно, это слишком большой макрообъект.
Аноним 22/08/21 Вск 01:04:23 675048160
>>675047
> А чёрной дыре всё равно, это слишком большой макрообъект.
Сделаем гв помощнее, чд поменьше, хоть на 1000 порядков (ну ок, не до планковских величин). Или какие-то принципиальные границы есть?
Аноним 22/08/21 Вск 03:10:47 675055161
>>675048
Нет, Хокинга всё равно будет корёжить, а чд нет. Хокинг величина, чд - глыба!
Аноним 22/08/21 Вск 03:12:21 675056162
>>675025
А тот шизик который про детонирущий Юпитер трактаты писал, где он?
Аноним 22/08/21 Вск 04:09:36 675060163
Сколько негров нужно, чтобы закидать ими черную дыру размером с ноготок мизинчика до первого выжившего негра?
Аноним 22/08/21 Вск 07:38:05 675075164
Аноним 22/08/21 Вск 13:53:10 675108165
>>672631 (OP)
Как в дирижаблях боролись с утечкой гелия?

Уточнение. Есть лёгкий корпус с обшивкой для защиты, внутри плотно расположены некие шары с гелием.
Аноним 22/08/21 Вск 17:38:53 675131166
>>675075
Нужно столько негров чтобы из них можно было Сатурн собрать или че?
Аноним 22/08/21 Вск 17:51:15 675132167
Аноним 22/08/21 Вск 18:32:45 675142168
Аноним 22/08/21 Вск 21:17:25 675158169
>>674871
Так чего, посмотрел, узнал первоисточник, прочитал в оригинале и готов обоссать моченого?
Аноним 22/08/21 Вск 22:28:47 675163170
изображение.png 621Кб, 600x400
600x400
изображение.png 898Кб, 742x488
742x488
изображение.png 5537Кб, 1689x1267
1689x1267
Почему говорят, что разрешение 20-30 см на пиксель это предел для спутников? Почему нельзя вхуярить большие линзы и матрицы на 100+ мегапикселей и получить разрешение больше? Чем ограничено это?
Аноним 22/08/21 Вск 22:31:25 675164171
Аноним 22/08/21 Вск 22:50:46 675171172
>>675164
В смысле то же, что и с телескопами, когда картинка плывёт? А коррекцию этого не делали, как с телескопами на земле?
Аноним 22/08/21 Вск 23:07:18 675173173
>>675163
1. Искажения атмосферы. Есть два способа с ними бороться - адаптивная оптика, и летать группой и делать композитный снимок с суперразрешением. Адаптивная оптика крайне затруднена на спутнике ДЗЗ - что униморфные с пьезоматрицей, что сегментные зеркала очень сложны, а лазерные корректоры требуют калибровочного источника, что применимо на телескопе, но не применимо на спутнике ДЗЗ, смотрящем на Землю. Полёты группой сами по себе сложны и имеют кучу подводных камней. Лаки имаджинг же слабо применим, т.к. спутник не делает мгновенных фоток, он непрерывно сканирует поверхность, как бумажный сканер.

2. Ширина полосы охвата. Нельзя бесконечно наращивать разрешение сенсора, и если ты увеличиваешь поверхностное разрешение, то приходится брать меньший угол обзора. В результате спутник сканирует узкую полосу, и ему нужно гораздо больше времени чтобы покрыть всю поверхность, оперативность съёмки сильно страдает.
>100 мегапикселей
В спутниках ДЗЗ используют не матричные ПЗС сенсоры, а линейные, с TDI (time delay and integration, временная задержка с накоплением заряда, ВЗН). Они похожи на матричные, но у них столбцы связаны в своего рода сдвиговой регистр. Спутник летит с определённой скоростью, а заряд перетекает между столбцами в обратном направлении, компенсируя это. В результате один кусок земли экспонируется по очереди всеми пикселями в строке, многократно увеличивая эквивалентную выдержку. Это как если бы там стоял линейный сенсор от бумажного сканера, но не с точечными пикселями, а очень сильно вытянутыми.

3. Полоса пропускания. Спутник и так генерит овердохуища данных. При увеличении разрешения и ширины охвата тебе придётся хранить охуительные объёмы на борту, и скидывать их на Землю. Производительность абсолютно всех современных спутников ДЗЗ ограничена полосой даунлинка до наземных станций. Чтобы её увеличить, надо строить больше наземных станций, чтобы пролёты были чаще. Возможно релейные сети вроде Старлинка улучшат ситуацию, но пока об этом не слыхать.
Аноним 22/08/21 Вск 23:18:22 675175174
>>675158
ща, погодь сек
откуда инфа что сурдин жополаз?
Аноним 22/08/21 Вск 23:46:27 675177175
АААХАХАХ soooquaaa как же лоллирую с дурачков
>Сурдин: вот я-то что узнал, ой что узнал оййй! хотят вернуть с титана грунт, а горючее возьмут из лужи. Вот уж никогда не думал что горючее можно взять из лужи, это всё с ног на голову переворачивает!
Ох пиздец, "никогда он не думал", а что для горючего нужен окислитель тоже ты не думал? А сколько нужно оксислителя не, не прикинуть? А почему я об этом сразу же>>674809 подумал и всё нахуй прикинул? а? ААА??!
>лезу читать на англ.
>https://www.space.com/amp/saturn-moon-titan-sample-return-mission
Ммм! Давай же, удиви меня!

>And with the landing out of the way, the next big challenge would be to get the sample up to space, on its way back to Earth. For that, the Glenn team envisions, the surface lakes of methane will come in handy. "Producing rocket fuel on Titan wouldn't require chemical processing — you just need a pipe and a pump," said Oleson. "The methane is already in a liquid state, so it's ready to go."

ВООУУУУУ! Вот это энвижен, вот это откровение, вот это mastermind! Совсем не нужно chemical processing, ты просто pump из лужи, вот-это-да
Но постой-ка, этот genius ещё не закончил

>The Compass Lab team will now investigate how to effectively produce liquid oxygen, to enable the fuel to burn. One option is to melt Titan's "rocks" of water ice with a nuclear heat source and then split the water.

ВОУ-ВОУ ПОЛЕГЧЕ ДРУГ, EASY! Ты и правда хочешь привезти туда мазафакин ядерный реактор и электричеством разложить воду? На водород и кислород? И выпустить весь водород наружу чтобы вместо него накачать из лужи метан. MAN YOU CRAAAZY! THAT'S THE SPIRIT! AMERICA!!!
Какой же ебаный кринж, это просто пиздец. И этим ребятам дали 120 тысяч, годовую зарплату инженера или 2 годовых зарплаты инженера SpaceX чтобы что, чтобы год писать отчёт про то что выпускать водород наружу всё же нерентабельно?
Аноним 23/08/21 Пнд 00:14:53 675179176
>>675173
>т.к. спутник не делает мгновенных фоток, он непрерывно сканирует поверхность, как бумажный сканер
>В спутниках ДЗЗ используют не матричные ПЗС сенсоры, а линейные, с TDI
А если использовать КМОП матрицу, как в фотоаппаратах? Ведь тот же Isocell HM2 запускали на спутнике
https://www.ixbt.com/news/2020/03/10/108-xiaomi-mi-10-pro.html
Да и я очень сильно сомневаюсь, что у Dove и Flock стоят TDI, а не матрицы, иначе мы бы не видели фоток кораблей в океане и других движущихся объектов
>делать композитный снимок с суперразрешением
Опять же, если мы запустим матрицу, что мешает сделать несколько снимков и обмазываться всякими субпиксельными сдвигами и deep fusion?
>При увеличении разрешения и ширины охвата тебе придётся хранить охуительные объёмы на борту, и скидывать их на Землю
Смартфоны при энергопотреблении в пару десятков ватт вполне могут обрабатывать снимки. А на спутник так вообще можно поставить что-то вроде Nvidia Jetson и обрабатывать на месте. Почему это не развивают? Тут вообще никаких принципиальных препятствий нет, а большинство тянут сырые данные со спутников почему-то
Аноним 23/08/21 Пнд 15:14:25 675249177
>>675179
>Да и я очень сильно сомневаюсь, что у Dove и Flock стоят TDI, а не матрицы, иначе мы бы не видели фоток кораблей в океане и других движущихся объектов
У планет лабс даже в спеках указаны TDI. Движущиеся объекты как-то снимают с компенсацией, это по факту не проблема. А вот как они фоткали старты спейсикса, я хз, это мне уже чёрной магией кажется, надо бы разобраться. (может быть в сенсоре есть более короткий путь, хз). Но у них 100% линейные детекторы, как и у всех, никто в здравом уме не будет фоткать одним кадром, ибо выигрыш у TDI перед мгновенными фотками феноменальный, тем более для планетовских мелкомыльниц.
>Опять же, если мы запустим матрицу, что мешает сделать несколько снимков и обмазываться всякими субпиксельными сдвигами и deep fusion?
Мешает тот факт, что в TDI детекторе происходит реальный физический перенос заряда и реальная выдержка. А программный стак из N фоток с выдержкой t эквивалентен всего лишь sqrt(N)*t выдержке.
>Смартфоны при энергопотреблении в пару десятков ватт вполне могут обрабатывать снимки. А на спутник так вообще можно поставить что-то вроде Nvidia Jetson и обрабатывать на месте. Почему это не развивают?
Не очень понял что ты собрался обрабатывать и во что преобразовывать. Всё и так стакается безо всяких GPU, бутылочным горлом являются реальные данные, которых просто тупо настолько дохуя что они еле успевают передаваться через имеющиеся наземные станции.
Аноним 23/08/21 Пнд 15:35:28 675251178
изображение.png 105Кб, 520x322
520x322
>>675249
>Всё и так стакается безо всяких GPU, бутылочным горлом являются реальные данные, которых просто тупо настолько дохуя что они еле успевают передаваться через имеющиеся наземные станции.
Если у нас спутник делает несколько снимков одного места, то можно склеить их в один прямо на спутнике и передать один снимок, а не 10
>Мешает тот факт, что в TDI детекторе происходит реальный физический перенос заряда и реальная выдержка
Всё равно не понял. Выдержка ж наоборот минимальная нужна, чтоб резкий снимок был. Ну можно ставить не мыльницы из смартфонов, а что-то типа пикрила с огромными пикселями
https://www.dpreview.com/articles/7964414898/canonlargestsensor
Аноним 23/08/21 Пнд 16:04:45 675255179
>>675251
>Если у нас спутник делает несколько снимков одного места, то можно склеить их в один прямо на спутнике и передать один снимок, а не 10
Можно, но TDI лучше.
>Выдержка ж наоборот минимальная нужна, чтоб резкий снимок был.
TDI как раз комбинирует лучшее из обоих миров. Один и тот же участок земли последовательно выдерживается на каждом пикселе строчки сенсора. И получается реальная выдержка, а не софтовый стек.
Движущиеся объекты там как-то по своему сшиваются прямо на месте, чтоб не артефачило - вот это я не в курсе, там куча магии в которой надо разбираться.
Аноним 23/08/21 Пнд 17:48:06 675270180
>>675255
Есть же стабилизациядвижением матрицы. Немного доработать алгоритм - и вот твой сенсор уже компенсирует орбитальное движение спутника. Пускай ненадолго, но и доработки минимальные.
Аноним 23/08/21 Пнд 23:45:18 675336181
>>675270
В результате получится куда худшая чувствительность всё равно, ибо нет физической связи между пикселями, и каждый сдвиг происходит полный ридаут (и соответственно сброс заряда). Вмешивается шум чтения. Поэтому ВНЗ всегда будет в разы лучше любых 2D матриц - заряд перетекает вдоль всего сенсора без считывания, а шум чтения попадает только в финальный результат, а не во все промежуточные.
Аноним 24/08/21 Втр 07:22:16 675358182
>>674719
Ну так и 1000 км сверхплотной материи - тоже дохуя так-то. Чье де дохуя дохуевее? Как прикинуть хотя бы порядок?
Аноним 24/08/21 Втр 18:02:47 675505183
1) Придумался концепт такой планеты, у которой масса распределена ненравномеро, из-за чего ядро расположено не по центру, а, например, на 3/4 сдвинуто к поверхности от географического центра планеты в направлении экватора. Планета вращается медленно, поэтому формы яйца у планеты не будет. но как это скажется на самой планете, на планетарное магнитное динамо и на орбиты потенциальных лун?
2) Еслть какие-нибудь сайты, где можно онлайн посмотреть красивые планеты? То есть, те или иные миры в худосжественном восприятии, придуманные миры, миры из игр и т.д., но обрисованные приемлемо, реалистично или К Р А С N В 0?
3) Почему у нас планеты вращаются в целом по круговой орбите (уж Земля-то точно), а по явно вытянутой - почти нет? имеются ли зафиксрованные звёзды с подтвержёднной планетарной системой, где планеты вращаются по вытянутым орбитам?
Аноним 24/08/21 Втр 19:26:43 675524184
KepkkoyC4c.jpg 249Кб, 485x663
485x663
>>673092
> Ёбнула ебическая ёба с плотностью сотни килограмм или несколько тонн на см3
> Какая защита потребовалась бы
Никакая, за 1000 км от такой хуёвины испаришься ещё на подлёте ещё до взрыва.
Аноним 25/08/21 Срд 09:31:20 675663185
16279879710480.jpg 212Кб, 826x1169
826x1169
Как вкатиться в разработку аэрокосмических аппаратов и систем?
Аноним 25/08/21 Срд 09:35:39 675666186
>>675177
Ты в курсе, что кислород на Титане будет жиденьким, а водород - все так же газообразным под любым давлением?
Ведь любому анонимному эксперту с двачей очевидно же, что система терморегуляции для ЖВ работающая годами с минимальным процентом потерь в условиях вчетверо более плотной атмосферы чем на Земле будет проще, легче и дешевле чем система качающая жидкость из лужи.
Аноним 25/08/21 Срд 14:00:06 675704187
Че будет если надеть на голову герметичный шлем с подачей воздуха и выйти жарким днем погулять по марсу без костюма?
Аноним 25/08/21 Срд 14:09:18 675713188
image.png 126Кб, 220x308
220x308
Допустим, чисто теоретически, у меня есть пара миллиардов долларов.
Я могу создать в России свою ракетно-космическую корпорацию? Или есть какие-то фундаментальные причины, по которым этого не получится сделать?
Аноним 25/08/21 Срд 14:10:45 675714189
>>675713
> Допустим, чисто теоретически, у меня есть пара миллиардов долларов.
> Я могу создать в России свою ракетно-космическую корпорацию? Или есть какие-то фундаментальные причины, по которым этого не получится сделать?
Бутылка
Аноним 25/08/21 Срд 14:11:28 675715190
Аноним 25/08/21 Срд 14:16:53 675718191
1629890210610.png 194Кб, 1079x816
1079x816
Аноним 25/08/21 Срд 14:47:16 675742192
00249psisdg1018[...].jpg 58Кб, 503x286
503x286
20210825142819.jpg 40Кб, 425x768
425x768
>>675666
челидзе, о каком "минимальном проценте потерь" ты говоришь? не будет никакого процента потерь.
вот как это работает: сначала ядерный реактор(!) подаёт питание в электролизер, куда уже загружен образец грязного льда, добытый сложным грунтозаборным устройством с манипуляторами и пилами(!). всего нужно извлечь/загрузить/обработать 40(!) кубометров льда (а то и все 100, если он грязный). потом происходит растапливание льда, очистка воды, электролиз и выходит газообразный водород. водород охлаждается в теплообменнике до 100 К и поступает в конденсатор, откуда уже понемногу стекает в бак. так вот, этот самый конденатор это всего лишь небольшой бак, на одной из стенок которого натыканы криокулеры по типу пикрила. обрати внимание на массу, энергопотребление и ресурс. собственно, за 1 год нацедить 5т жидкого водорода вполне можно, весить эта система будет килограмм 150 (включая бак-теплообменник), потреблять 3-4кВт, притом что на электролиз нужно 25-30 кВт. Хочешь заправлять метан? Ок, тебе нужно всё то же что и для водорода, минус небольшой бачок-конденсатор, не сильное-то упрощение, а? ресурс криокулеров, напомню, 200 тыс. часов или 23 года.
В общем так, сынидзе. Я анонимный эксперт, я знаю что говорю. А вот ты всего лишь ещё один кукарекающий "контраргументы" двачан, нахватавшийся по верхам популярной хуйни типа "водород всегда выкипает". хрр-тьфу
Аноним 25/08/21 Срд 14:47:57 675744193
>>675713
Ну начнем с того, что все ракеты кроме совсем уж любительской мелочевки относятся к технологиям двойного назначения, поэтому или ты сам сперва придешь на поклон к siloviki или они придут к тебе. При этом с нашим размахом шпиономании и любви к поиску внутренних врагов, в сочетании с параноидальной привычкой секретить все что только можно и нельзя, даже если siloviki будут к тебе благосклонны, получишь ты из своей конторы режимное предприятие, с сейфами для мобильников на проходной, и постоянным мониторингом на тему а не продаешь ли ты свои секретные технологии ворогам заграничным. Но это если они будут благосклонны, а они не будут, потому что прибыли что бы тебя стричь у тебя не будет очень долго, и проще тебя сразу набутылить, так хоть палка будет и может лишняя звездочка. Все равно делать свои ракеты ты будешь не совсем с нуля от работ Годдарда и Циолковского, так что найти что именно ты пытаешься бесплатно продать американцам с китайцами смогут.
Аноним 25/08/21 Срд 15:19:45 675763194
>>675744
>относятся к технологиям двойного назначения,
Технологии и реальное применение технологий все таки разные вещи. Да и за что бутылить? Конкретная статья какая? Если я никаких договоров с силовиками не заключал, я обычный бизнесмен с гражданским бизнесом.
Аноним 25/08/21 Срд 16:05:33 675782195
>>675742
Чел, криокулер который ты притащил вообще не пригоден для жидкого водорода. Но в любом случае, тебе необходима криосистема для ЖВ, необходим огромный теплоизолированный бак.
В то время как для метана на Титане вообще не нужно охлаждение и теплоизоляция, его в обычной бочке там хранить можно. Ровно как и для кислорода, ему максимум что нужно - чутка повышенное давление в баке.
И что бы не играть в анонимного эксперта, я прямо цитату из обсуждаемой статьи принесу.
>In our baseline system, the water electrolysis will produce hydrogen that is vented as surplus, but an alternative approach would be to use a LOX/LH2 engine, at >430 s Isp. This is not likely to be the optimum solution for the booster stage required to launch from the surface, because of the low density and difficulty of storage, but could be a possible high-Isp choice for one or both of the smaller Earth return stages, for which specific impulse becomes most important. However, storing the LH2 is more difficult, and would require a thermally insulated storage system and a cryocooler to keep the hydrogen liquid, at about 30K.
Аноним 25/08/21 Срд 16:08:26 675784196
Смогут ли негры колонизировать Меркурий?

У них темный цвет кожи и резист к ожогам, значит у них есть шансы.
Аноним 25/08/21 Срд 16:11:28 675786197
>>675784
Можно использовать негров вместо теплозащитных плиток Starship.
Аноним 25/08/21 Срд 16:15:08 675788198
Сколько негров нужно чтобы облепить ими старшип вместо плитки? Целесообразно ли использовать негров вместо плитки?
Аноним 25/08/21 Срд 16:18:26 675790199
>>675788
Не забывай, что у негров расход в 3 раза больше. Так что придется лепить в 3 слоя.
Аноним 25/08/21 Срд 16:29:01 675796200
>>675763
Есть нормы экспортного контроля, которые распространяются на фирмы, работающие со всеми технологиями, сырьем и оборудованием из ряда списков (ракеты, ядерка, некоторый химпром и т.д), за нарушение которых предусмотрена в том числе и уголовная ответственность. При этом нормы относятся не только к конечным продуктам, а к большому количеству промежуточных результатов, включая информацию:
>контролируемые товары и технологии - сырье, материалы, оборудование, научно-техническая информация, работы, услуги, результаты интеллектуальной деятельности (права на них), которые в силу своих особенностей и свойств могут внести существенный вклад в создание оружия массового поражения, средств его доставки, иных видов вооружения и военной техники
Вот просто, если ты работаешь с чем-угодно из соответствующих списков (для ракет вот: https://fstec.ru/component/attachments/download/1737), то просто за то что ты что-то выложил в интернете тебя можно набутылить. А определять, действительно ли инфографика с официального сайте по которой можно понять что у тебя ракета, у нее есть двигатели и даже тип топлива указан, будет уважаемый эксперт. Вот подтвердит он, что ты сообшил иностранцам что ракете нужен двигатель, и это будет существенным вкладом в создание оружия массового поражения, и доказывай суду что это не так.
А уж если ты планируешь свою деятельность не ограничивать исключительно российским внутренним рынком, то вот вообще.
Аноним 25/08/21 Срд 16:30:12 675798201
Аноним 25/08/21 Срд 16:46:34 675801202
>>675796
>просто за то что ты что-то выложил в интернете
А если я не выкладываю ничего в интернете?
Да и вообще, если я сам придумал свою технологию, ничего не использовал и тем более не брал ничего от российского государства, то меня всё равно обутылят? Если да, то попахивает тоталитаризмом.
Аноним 25/08/21 Срд 16:57:00 675803203
>>675801
Если ты делаешь что-то на основании чего можно сделать, например, МБР, и результаты твоей деятельности могут быть получены иностранцами, то да, тебя набутылят. Гипотически, если ты нигде не распространяешься о своей деятельности, работаешь исключительно на внутренний рынок и не обращаешься к иностранным подрядчикам, то набутылить за распространение не смогут. Вот только если ты при этом не обеспечиваешь требования прозрачно для силовиков и в соответствии со всеми стандартами, то вполне достаточно будет "обоснованного подозрения". Дальше маски шоу, вывоз всей техники на экспертизу, заключение на время расследования и т.д. А там найдут, что кто-то из твоих сотрудников в фейсбуке похвастался, что двигатели делает.
Аноним 25/08/21 Срд 17:01:03 675804204
Аноним 25/08/21 Срд 17:05:06 675805205
>>675663
Только получить ВО и пытаться устроиться

Либо техником и допрашивать потом инженеров
Аноним 25/08/21 Срд 17:06:40 675806206
>>675704
Удвою вопрос!

Подозреваю, что за час прогулки ничего ужасного, хотя нет, там же атмосфера иначе Солнечные лучи пропускает, так что могут быть и ожоги.
Аноним 25/08/21 Срд 18:37:55 675824207
>>675782
> at about 30K
Эти ебанаты тоже нихуя не шарят в криотехнике? Мдээ...всё это грустно. Походу учёнишки уровня одной тупой милфы, которая опубликовала статью про источники энергии на Титане, и там блять не было химической ТЭЦ, не было соответствующего топливного цикла (а он есть, я его разработал, например), а была абсолютно дегенеративная поебень "если мы сделаем на Титане поле солнечных батарей размером с США, то выработка кВт будет как раз как у современных США, это отличная идея". Солнечные блять батареи на ебучем Титане где 1/1000 земного освещения на поверхности? В рецензируемой статье?! От учёного из центра стениса-хуениса? Что же с нами стало, боги!
по теме: при 30К плотность жидкого водорода вдвое меньше чем при 20К, только дегенерат будет такое предлагать. ну или совсем мимо чувак, не шарящий, прям как ты, который нихуя не понимает за криокулеры Стирлинга и подходят они для водорода или нет. утомил ты меня, прощай.
Аноним 25/08/21 Срд 18:41:59 675825208
>>675704
С голым телом? Не сможешь дышать, нужно хотя бы корпус закрыть скафандром. А руку/ногу можно оголить, ничё не будет. Минут через 5-10 рука сильно распухнет, но зайдёшь в дом всё сдуется обратно. Под кожей будут появляться-исчезать пузырьки водяного пара, вероятен некоторый зуд.
Кстати то же самое можешь делать и на Луне. А вот на Титане вообще голым сможешь выскочить, потрясти хуишком, поорать матюки, обмораживаться начнёшь только через минуту-полторы.
Аноним 25/08/21 Срд 19:42:46 675834209
>>675704
Если оголишься целиком - потеряешь сознание и сдохнешь. Это не вакуум, но разница не настолько большая.

Если оголишь часть тела - пидор синяки гематома кладбище.

Ну и плюс ты охуеешь давление 1 атм лёгкими держать. Если только разреженным кислородием дышать, как на аполлоне.
Аноним 25/08/21 Срд 20:02:07 675837210
>>675824
Ты уже написал в SpaceX про космическую радиацию? А то похоже эти идиоты о ней забыли.
Аноним 25/08/21 Срд 20:32:04 675847211
>>675825
>А вот на Титане вообще голым сможешь выскочить, потрясти хуишком, поорать матюки, обмораживаться начнёшь только через минуту-полторы.
В криосауне на максимальной заморозке -170C дольше двух минут не держат. Причём ебло находится строго снаружи, чтобы холодный воздух в лёгкие не попал. Но на Титане атмосфера в 4 раза плотнее, так что даже в шлеме уже через 30 секунд ты словишь обморожение. Без шлема тебе гарантированный пиздец сразу.
Аноним 26/08/21 Чтв 01:05:46 675889212
>>675505
1) Такая планета нереалистична. На этапе образования, когда все расплавлено нахуй (не забывай, что там всякое говно слепляется в одну кучу, и кинетическая энергия падающего говна переходит в тепловую, так что такой этап обязательно был, если у тебя не астероид 10-20 км), вещество под действием самогравитации распределится более-менее равномерно. Подозреваю, что даже если взять искусственную и нерастопленную планету размером с Землю, где такое распределение масс, там все быстренько растопится довлением и начнет течь.
Если гипотетически: динамо хз (один хуй до сих пор нет нормальной теории), лунам практически похуй, разве что приливный захват должен быстрее возникать, как мне кажется.
3) Видимо, так получилось. Да, имеются.
Аноним 26/08/21 Чтв 01:51:50 675892213
>>675784
Можно построить подземный бункер в районе одного из полюсов, там вроде даже есть кратеры, куда не заглядывает солнушко. В таком даже беляши выживут, наверное.
Аноним 26/08/21 Чтв 07:05:37 675907214
>>675505
>Почему у нас планеты вращаются в целом по круговой орбите (уж Земля-то точно), а по явно вытянутой - почти нет?
Потому что всё что могло - посталкивалось на раннем этапе или эволюционировало, влияя друг на друга. Куче массивных тел на пересекающихся эллиптических орбитах в одной плоскости довольно сложно образовать устойчивую систему.
>имеются ли зафиксрованные звёзды с подтвержёднной планетарной системой, где планеты вращаются по вытянутым орбитам?
Малые и далёкие транснептуновые тела, где этот эффект меньше, вполне себе крутятся по высокоэллиптическим орбитам, прямо в нашей системе - например Плутон или Седна. В других бывают и большие планеты такие. Про них пока слишком мало известно на практике, мы видим в основном большие газовые гиганты или суперземли, и то еле-еле. Но эффект скорее всего тот же - если планет мало и звезда молодая, то они вполне могут и не собраться в круги. Или ещё какие экзотические условия могут быть.
Аноним 26/08/21 Чтв 07:52:07 675908215
>>675892
В Меркурий-треде всё это было. Но Меркурий-треда больше нет.
Аноним 26/08/21 Чтв 07:54:36 675909216
>>675847
>в криосауне
>на Титане
>в 4 раза плотнее
ну и дебс! кошмар!! ди нахуй
Аноним 26/08/21 Чтв 08:14:20 675912217
>>675505
> где планеты вращаются по вытянутым орбитам?

Нибиру же. С аннунаками. шутка
Аноним 26/08/21 Чтв 09:15:12 675916218
>>675834
>Ну и плюс ты охуеешь давление 1 атм лёгкими держать.
Почему легкими? Каркас тела - ребра, позвоночник. А легкие внутри.
Аноним 26/08/21 Чтв 09:26:28 675917219
>>675916
Без разницы, дышать нормально так ты не сможешь всё равно, разопрёт
Аноним 26/08/21 Чтв 15:34:34 676020220
>>672631 (OP)
ИЩУ ТРЕД С ЧЕРТЕЖАМИ
пару лет назад находил тут тред чуть ли не с 500 классными и редкими картинками чертежей внутри. если у кого осталась ссылка на него или на новые с такой же тематикой - поделитесь пожалуйста
Аноним 26/08/21 Чтв 15:36:51 676021221
>>675825
АНОН СОВСЕМ ЗАБЫЛ ЧТО ЧЕРЕЗ ОЧКО БУДЕТ ВОЗДУХ ВЫЛЕТАТЬ
и через хуй моча
Аноним 26/08/21 Чтв 16:05:19 676036222
>>676020
Наверно самый тупой вопрос за последние пару лет. >>168735 (OP)
Аноним 26/08/21 Чтв 16:23:06 676038223
>>676036
СПАСИБО ТЕБЕ АНОНЧЕ
Аноним 26/08/21 Чтв 17:50:48 676056224
>>675825
Не выдумывай про Титан, обморозишься сразу, там плотность выше в четыре раза (давление в полтора, сам знаю, но плотность в 4), и температура ниже чем в антарктиде больше чем вдвое, если в кельвинах считать
Аноним 26/08/21 Чтв 20:06:40 676080225
>>676056
пшла вон тупая жирная псина
Аноним 27/08/21 Птн 02:11:40 676163226
>>676056
А если сперва в баньку, чтоб распариться?
Аноним 27/08/21 Птн 08:05:13 676186227
>>676163
это поможет. а после баб ебать. скажи титан круто
Аноним 27/08/21 Птн 19:05:58 676319228
16282890568680.png 300Кб, 638x706
638x706
Если у Земли появится второй спутник, типа Минмуса у Кербина из КСП, и окажется на орбите
1) за Луной
2) между Землёй и Луной
как это скажется на Минмусе? А на Луне? А на Земле?
Аноним 27/08/21 Птн 20:05:37 676330229
>>676056
Человеческое тело намного лучше переносит высокое давление, чем низкое.
Аноним 27/08/21 Птн 22:38:34 676364230
>>676319
Зависит от массы спутника же. Возможны минмусовские приливы. Определенные конфигурации будут нестабильны в долгосрочной перспективе, вплоть до выбрасывания какого-то спутника нахуй. С другой стороны, если считать, что он "всегда там был", а не просто появился по щучьему велению, то конфигурация таки будет стабильной.
Аноним 27/08/21 Птн 23:27:51 676375231
Если курнуть гашиша на мкс можно сойти с ума ?
Аноним 28/08/21 Суб 04:08:30 676391232
>>676375
> Если курнуть гашиша на мкс можно сойти с ума ?
Откуда гаш на мкс?

Русня только коньяк протащить может. Гаш только в очке пронести, но тогда места для коньяка не останется.
Аноним 28/08/21 Суб 07:20:55 676402233
1630124453394.jpg 7Кб, 200x200
200x200
Аноним 28/08/21 Суб 11:26:25 676427234
16289820123040.png 209Кб, 544x400
544x400
>>676364
Я просто помню, кто-то как-то спросил про планету Меланхолию (как в одноимённом фильме), мол, что бы было бы если бы такая приблизилась к Земле и встала на орбиту вокруг Земли, то Меланхолию, как более лёгкое тело выпнет прочь.
Это как если бы Земля, Венера и Марс с Меркурием вращались бы вокруг Юпитера вместо галилеевских спутников, то, в зависимости от из взаимного расположения, более лёгкие Марс с Меркурием вышвырнуло бы.
Объяснялось так: более лёгкая планета-луна будет вышвырнута с орбиты, окажись она зажатой между более массивными планетами-лунами и самим Юпитером.
Мне это кажется несколько странным, потому что ведь можно же поставить тела в резонанс.
Аноним 28/08/21 Суб 16:02:47 676485235
>>676427
А кто спросил, кто ответил? Очередная болтовня невежд?
Аноним 28/08/21 Суб 17:33:43 676500236
>>676485
>кто спросил,
Аноним
>кто ответил
Аноним
Аноним 28/08/21 Суб 22:08:23 676535237
>>676500
Так я и думал. Вот что я тебе скажу: большая планета действительно может раздрочить маленькую настолько, что у той снесёт нахер башку и маленькая уебётся об звезду/большую планету лишь бы прекратить этот ебаный марафон безумной дрочки. Но тут всё зависит от конкретной ситуации. Скажем, спутникам Сатурна норм, хоть их там очень много и они все ничтожные кривые унтерспутники по сравнению с невероятным альфачом ТИТАНОМ. Даже мелкий педик Гиперион ещё на месте. Хотя есть некоторые основания полагать что Титан им таки показал, всех разъебал, кого-то съел нахуй, а педик Гиперион выжил чисто по случайности https://ui.adsabs.harvard.edu/abs/2014DDA....4530101C/abstract
Большой пидор Ганимед тоже ничего не делает писечке Еаропе, хотя та в разы легче Ганимеда. И писечка Ио на месте, стонет во все щели от того что папа жмёт, но держится. Спутники Плутонуса не улетают никуда, хотя у них там вовсю орудует Харонус. Не всё так однозначно, друг.
Аноним 28/08/21 Суб 22:10:56 676536238
>>676535
Харонус ебал Плутонуса в Sputnik Planum
Аноним 28/08/21 Суб 22:11:53 676537239
>>672631 (OP)
Скажите пожалуйста, когда полетит Луна-26?
Аноним 28/08/21 Суб 22:23:56 676538240
>>676537
Луна-26 полетит вслед за Луной-25. А Луна-25 полетит уже скоро, всё схвачено, надо только не бухтеть.
Аноним 28/08/21 Суб 22:44:15 676541241
>>676538
>надо только не бухтеть.
А то что?
Аноним 29/08/21 Вск 10:32:35 676606242
20210829103208.jpg 79Кб, 1280x720
1280x720
Харонус ебал Плутонуса в Sputnik Planum
Аноним 29/08/21 Вск 10:34:30 676607243
>>676538
Луна-25 полетит в мае и сядет успешно без серьезных проблем во время перелета. Запомните этот твит. Инфа 80%.
Аноним 29/08/21 Вск 11:44:32 676635244
20210829114131.jpg 1834Кб, 2730x2048
2730x2048
20210829114401.jpg 88Кб, 666x720
666x720
Харонус ебал Плутонуса в Sputnik Planum
Аноним 29/08/21 Вск 15:01:39 676658245
20210829145755.png 741Кб, 536x660
536x660
Sputnik Planum.jpg 197Кб, 900x704
900x704
Чё, пацаны, Sputnik Planum?
Аноним 29/08/21 Вск 15:15:54 676660246
>>676607
В мае какого года?
Аноним 29/08/21 Вск 15:22:00 676664247
>>676658
Да. Именно туда Харонус ебал Плутонуса.
>>676660
через год летит а потом опять
Аноним 29/08/21 Вск 15:28:38 676666248
>>676658
Я вижу огромное количество неприличных рисунков тут
Аноним 30/08/21 Пнд 11:10:37 676779249
>>676538
Ну я полюбому буду кипеть в Октябре 2021, когда по старому графику должна была лететь "двадцать пятая".

Но чего они тянут с двадцать шестой? Там же простой орбитальный аппарат.
Аноним 30/08/21 Пнд 11:44:42 676791250
20210830113559.png 441Кб, 1017x767
1017x767
20210830114334.jpg 124Кб, 900x612
900x612
>>676666
Сложно скрыть огромный срам, когда ты размером с планету! Но там у Плутонуса до сих пор мягенько, учёные утверждают что Sputnik Planum это огромная пробоина заполненная вязким тёплым медленно конвектирующим воскообразным "льдом". Харонус таки торкнулся наотлично.
Аноним 30/08/21 Пнд 15:26:09 676851251
>>676791
Хуя там бородатые планетологи уже начертили идей
Аноним 30/08/21 Пнд 16:35:25 676871252
Аноним 30/08/21 Пнд 16:52:49 676879253
1gfFOhpbGOq2l94[...].jpeg 123Кб, 960x585
960x585
>>676851
Ну кстати неплохо напридумывали они там
Аноним 30/08/21 Пнд 17:10:06 676885254
>>676879
Такая линза воды и на Европе предполагается, причём глубина залегания вполне может быть 2-3 км всего. А это значит что там можно жить. А если 20-30 км то нельзя там жить. Время покажет. Энцелад пока что смотрится перспективнее, там аж до самого каменного ядра можно добраться, и разрабатывать минералы. Прямо кирками и бурами, вручную, без скафандров. А на Европе до ядра не добраться, только телеуправляемыми аппаратами, что означает невозможность добычи.
Аноним 30/08/21 Пнд 22:53:17 677008255
>>676885
>А если 20-30 км то нельзя там жить.
Тьфу, молодежь пошла. В войну и не такое переживали, и ниче!
Аноним 31/08/21 Втр 06:11:04 677060256
>>676779
>Но чего они тянут с двадцать шестой? Там же простой орбитальный аппарат.
Да там похоже с ним еще вообще ничего не сделано, работать начнут только после запуска Луны-25. Это ясно уже из того как сроки переносятся, порой в формате "баллистические условия для посадки луны-25 неподходящие, поэтому 25 и 26 переносятся на год".
Это кстати значит, что между 25 и 26 тоже лет 5-10 пройти может.
Cupy 31/08/21 Втр 08:59:52 677081257
В переговорах слышу слово "cupy" (или cuby).
Похоже, что произносится это слово в ответ. Никаких других слов отвечающий не произносит.
Услышать можно тут (транса стыковки CRS-23 с МКС) https://youtu.be/TwbXHFJ8pdo?t=480
Что оно значит?
Аноним 31/08/21 Втр 10:09:34 677092258
>>677081
Это слово "copy" и означает "услышал и принял".
Аноним 31/08/21 Втр 14:20:20 677140259
Придумал как можно доставить грузы в космос. Надо запустить очень сильно надутый воздушный шар с кольцом на верху. Шар максимально высоко поднимается к космосу, а дальше с помощью лебедки приделанной к спутнику вытягивать шар в космос вместе с грузом крюком на тросе из каких-нибудь прочных алмазных сплавов.
Из-за того, что трос будет не очень длинный, он не порвется от своего веса, как если бы он вытягивал груз с поверхности земли, или даже большой горы.
Аноним 31/08/21 Втр 15:50:01 677183260
maxresdefault.jpg 72Кб, 1280x720
1280x720
crab-clipart-gr[...].jpg 140Кб, 1024x757
1024x757
>>676885
>Энцелад пока что смотрится перспективнее
А ты у нас спросил, обезьяна лысая? Будешь бежать с нашего Энцелада с киркой в жопе, чмо.
Аноним 31/08/21 Втр 17:40:35 677254261
20210831173946.jpg 56Кб, 460x686
460x686
>>677183
Спокойно, пацаны, давайте придём к компромиссу. Не надо перекусывать наши провода и шланги, изи, френд, изи!
Аноним 31/08/21 Втр 22:34:38 677336262
>>677140
>на верху
Идея неплохая, но бес граммотная.
Аноним 31/08/21 Втр 23:06:16 677341263
>>677336
значение знаешь?
Аноним 31/08/21 Втр 23:12:00 677342264
Протоны или альфа ядра солнечного ветра могут вызвать какие-либо ядерные реакции в реголите или атмосфере?
Аноним 31/08/21 Втр 23:16:45 677343265
Аноним 01/09/21 Срд 03:00:08 677347266
>>672631 (OP)
наука в тупичке, пока не признают эфир, даже официалы дошли умом что вакуум не пуст, и в нем что-то есть. Волны - возмущение в среде, распространяющееся в пространстве. С чего вы думаете что с ЭМ волнами все не так и они охуенно особенные
Аноним 01/09/21 Срд 05:17:51 677353267
>>677347
Вот боюсь забанят, а так бы дал ссылку на одного шизика-непризнанного-учёного. Там крыша сорвана так, что мама не горюй.
И бреда он нагенерил столько, что хватит на десяток безумцев.
Но самое забавное во всём этом что среди потока бреда он дал очень интересное объяснение гравитации. К сожалению я не могу сейчас найти - он очень плодотворен и рыться с мегабайтах чуши, чтобы найти то самое объяснение я не могу. Но красиво расписал. Если бы он вовремя остановился на этом, то вероятно даже нашёл бы последователей, но он ударился во все тяжкие и придумал свою периодическую систему элементов, замешал всё философией, социологией, происхождением языков, и всё это вместе не позволяет воспринимать его нормальным. Там реальная клиника и синдром "непризнанного гения".

Но при всём этом объснение гравитации через эфир красиво, стройно и понятно. Нет, я не его последователь, но гипотеза интересная. Будет забавно если физика повернётся к его теории.


Аноним 01/09/21 Срд 06:56:14 677358268
Аноним 01/09/21 Срд 08:55:10 677377269
Ок, читаем, что пишет Ленин про "материя исчезает":

Когда физики говорят: "материя исчезает", они хотят этим сказать, что до сих пор естествознание приводило все свои исследования физического мира к трем последним понятиям - материя, электричество, эфир; теперь же остаются только два последние, ибо материю удается свести к электричеству, атом удается объяснить как подобие бесконечно малой солнечной системы, внутри которой вокруг положительного электрона двигаются с определенной (и необъятно громадной, как мы видели) быстротой отрицательные электроны. Вместо десятков элементов удается, следовательно, свести физический мир к двум или трем (поскольку положительный и отрицательный электроны составляют "две материи существенно различные", как говорит физик Пеллà, - Key, l. c., р. 294-295[185]). Естествознание ведет, следовательно, к "единству материи" (там же)[186] - вот действительное содержание той фразы об исчезновении материи, о замене материи электричеством и т. д., которая сбивает с толку столь многих. "Материя исчезает" - это значит исчезает тот предел, до которого мы знали материю до сих пор, наше знание идет глубже; исчезают такие свойства материи, которые казались раньше абсолютными, неизменными, первоначальными (непроницаемость, инерция, масса и т. п.) и которые теперь обнаруживаются, как относительные, присущие только некоторым состояниям материи. Ибо единственное "свойство" материи, с признанием которого связан философский материализм, есть свойство быть объективной реальностью, существовать вне нашего сознания.

Ошибка махизма вообще и махистской новой физики состоит в том, что игнорируется эта основа философского материализма и различие материализма метафизического от материализма диалектического. Признание каких-либо неизменных элементов, "неизменной сущности вещей" и т. п. не есть материализм, а есть метафизический, т. е. антидиалектический материализм. Поэтому И. Дицген подчеркивал, что "объект науки бесконечен", что неизмеримым, непознаваемым до конца, неисчерпаемым является не только бесконечное, но и "самый маленький атом", ибо "природа во всех своих частях без начала и без конца" ("Kl. ph. Sehr.", S. 229-230[*187]). Поэтому Энгельс приводил свой пример с открытием ализарина в каменноугольном дегте и критиковал механический материализм. Чтобы поставить вопрос с единственно правильной, т. е. диалектически-материалистической, точки зрения, надо спросить: существуют ли электроны, эфир и так далее вне человеческого сознания, как объективная реальность или нет? На этот вопрос естествоиспытатели так же без колебания должны будут ответить и отвечают постоянно да, как они без колебаний признают существование природы до человека и до органической материи. И этим решается вопрос в пользу материализма, ибо понятие материи, как мы уже говорили, не означает гносеологически ничего иного, кроме как: объективная реальность, существующая независимо от человеческого сознания и отображаемая им.

Реакционные поползновения порождаются самим прогрессом науки. Крупный успех естествознания, приближение к таким однородным и простым элементам материи, законы движения которых допускают математическую обработку, порождает забвение материи математиками. "Материя исчезает", остаются одни уравнения.

Жаль, что 99,99% антикоммунистов не в состоянии понять этот текст, поэтому не поймут какую мочу им льют в уши. Ну да и пускай обтекают.
Аноним 01/09/21 Срд 10:19:21 677387270
>>677377
Ты ошибся разделом, /re двумя блоками ниже.
Аноним 01/09/21 Срд 10:19:35 677388271
>>677377
Котлеток наверни заместо Ленина, полегчает.
Аноним 01/09/21 Срд 11:57:14 677404272
>>677347
Таблетки прими, ёбаный ты шизопитек.
Аноним 01/09/21 Срд 11:58:22 677405273
>>677377
Ты в курсе, что Сралин ебал Членина в жопу, коммипидорок?
Аноним 01/09/21 Срд 15:21:45 677467274
Помогите понять гравитацию и предел Роша на таком примере: вот есть у нас стандартная Земля и стандартная Луна, стандартно вращающаяся вокруг Земли. Земля вдруг начинает увеличиваться в объёме, не изменяясь в массе и сотавляя каким-то неверотяным образом поверхность неповерждённой. Земля, вода, биосфера - всё на месте. Атмосфера при этом как было 100км, так и остаётся.
Как разбухшая Земля повляет на орбиту Луны и как вращение Луны повлияет на разбухшую Землю? Ведь в пропорциональном соотношении расстояние между Землёй и Луной сократилось.
Если изначальный радиус Земли очень грубо округляем до 7000км, а затем он разбухает до 14000км, до 70000км или до 150000км или вообще до 300000км?
Аноним 01/09/21 Срд 15:35:40 677475275
>>677405
Как что-то плохое.
Аноним 01/09/21 Срд 15:47:58 677480276
>>677475
Для коммиблядей ничего такого, для них долбиться в пердильник - в порядке вещей.
Аноним 01/09/21 Срд 16:06:52 677485277
Почему Боинг непрерывно обсирается?
Аноним 01/09/21 Срд 16:12:57 677486278
BoeingX-37Bafte[...].jpg 215Кб, 1792x1206
1792x1206
Аноним 01/09/21 Срд 18:17:05 677512279
>>677480
Это, вообще-то, сейчас модно. Не будь таким ретроградом.
Аноним 01/09/21 Срд 18:41:38 677519280
Так ли нужны люди на околоземной орбите и в космосе? Слишком же много для жизнедеятельности ресов возить, нужели там никак не автоматизировать.
Аноним 01/09/21 Срд 18:58:55 677523281
>>677512
Спасибо за предложение, но я лучше побуду ретроградом.
Аноним 01/09/21 Срд 19:08:39 677525282
Если бы на Юпитере (например на ядре, или где-то в толще, на каких-то плавающих островах), существовала разумная жизнь, с уровнем технологического развития примерно соответствующим современному человечеству, то каким образом они могли бы получать сведения об окружающем космосе? Каким образом они могли бы связаться с Землёй?
Аноним 01/09/21 Срд 19:08:45 677526283
20210829114131.jpg 1834Кб, 2730x2048
2730x2048
20210829114401.jpg 88Кб, 666x720
666x720
Sputnik Planum.jpg 197Кб, 900x704
900x704
20210830114334.jpg 124Кб, 900x612
900x612
>>677405
А ты в курсе что это доска /spc а, антисетикоммипетушок? Никакого Сталина и Ленина здесь нет. Харонус! Харонус ебал Плутонуса в податливый конвекционный Sputnik Planum.
Аноним 01/09/21 Срд 19:13:33 677529284
>>677525
Сведения получать через наблюдения из верхних слоёв. Или запуски метеоракет. Связываться с Землёй по радиоволнам с передатчика в верхних слоях. Но сами они никогда бы не смогли подняться и увидеть, корабль должен держать внутреннее давление в десятки тысяч атмосфер и быть раскалённым до пары тысяч градусов. Невозможно.
Аноним 01/09/21 Срд 19:15:31 677531285
>>677523
Ты против прогресса чтоле епта?
Аноним 01/09/21 Срд 19:15:47 677532286
>>677529
>Или запуски метеоракет
С такой гравитацией это реально?
Аноним 01/09/21 Срд 19:17:44 677535287
>>677532
Метеозонд в виде воздушного шарика. Раз уж у нас там живут юпитериане, то недостатка в охуенно прочных материалах не должно быть.
Аноним 01/09/21 Срд 19:21:33 677536288
>>677532
Метеоракета. На Земле метеоракете достаточно набрать скорость в 1,5 км/с. Сравни с 8 км/с орбитальной. На Юпитере обычная ракета будет метеоракетой, то есть как писюн Безоса сможет вылететь за атмосферу, поболтаться там минут 5 в невесомости (провести наблюдения, да) и обратно. Можно даже отстрелить 5-ступенчатую ракету на водороде с УИ 480 сек и попытаться добрать 30 км/с чтобы вывести 2 кг на орбиту. Короче, даже на химии можно, а с ядерным/термоядерным двигателем тем более.
Аноним 01/09/21 Срд 19:39:11 677542289
>>677535
>охуенно прочных материалах не должно быть.
Так разница давлений нужна в тысячи атмосфер, у нас то воздушные шарики максимум одну держат.
Юпитере обычная ракета будет метеоракетой, то есть как писюн Безоса сможет вылететь за атмосферу
Что-то я сомневаюсь на счёт ЗА атмосферу.
Аноним 01/09/21 Срд 19:40:13 677543290
>>677536
> НаЮпитере обычная ракета будет метеоракетой, то есть как писюн Безоса сможет вылететь за атмосферу
Что-то я сомневаюсь на счёт ЗА атмосферу.
Проебался с разметкой.
Аноним 01/09/21 Срд 20:52:28 677561291
>>677542
Ну смотри сначала поднимаешь ракету на уровень 5-0,5 бар (балуном/самолётом/атмосферной платформой), а потом уже оттуда и стартуешь. У Юпитера величина scale height ,в атмосфере всего в 2 раза больше земной, т.е набрать высоту ракете нужно будет всего километров 300, а земной метеоракете 150. В общем аэродинамические потери и гравитационные потери не будут сильно больше. Тяговооружённость сделай 1,8 а не 1,15 - и вот тебе снижение гравитационых потерь. А общую скорость (конечную) юпитерианской метеоракете нужно будет иметь всего в 2,5 раза выше чем у земной метеоракеты. Земная набирает 1,3 км/с -> юпетиреанская должна набрать около 3 км/с. Да это даже не ракета, это огрызок ракеты, это...первая ступень. Писюн чуть длинее и толще чем у Безоса. В-вотт
Аноним 01/09/21 Срд 21:55:19 677571292
>>677519
люди там как объект, а не субъект
Аноним 01/09/21 Срд 22:44:31 677579293
Аноним 02/09/21 Чтв 06:50:12 677604294
>>677377
>апелляция к авторитету
>авторитет — сифилисная, плешивая жертва сексуального насилия
Аноним 02/09/21 Чтв 07:18:06 677605295
Аноним 02/09/21 Чтв 13:18:23 677652296
https://habr.com/ru/news/t/389241/

То есть эта хуйня уничтожила все разумные цивилизации в радиусе 1000 световых лет от себя?
Аноним 02/09/21 Чтв 13:23:36 677653297
>>677652
> То есть эта хуйня уничтожила все разумные цивилизации в радиусе 1000 световых лет от себя?
Кого уничтожила? Ты жив пока что. И я жив.
Аноним 02/09/21 Чтв 13:24:30 677654298
>>677140
>Минимум 400км-30км=370км
Да да... Достаточно короткий трос.
Аноним 02/09/21 Чтв 14:53:59 677670299
>>677140
Космос - это не высота, а скорость. У шара скорость околонулевая.
Аноним 02/09/21 Чтв 14:57:17 677671300
>>677670
Ебать, это же будет почти геостационар, но на низкой орбите, и с возможностью возврата, охуенно же, что тебе не нравится?
Аноним 02/09/21 Чтв 14:59:08 677672301
На какой практической минимальной высоте над поверхностью Луны возможен орбитальный полет?
Аноним 02/09/21 Чтв 15:03:25 677673302
image.png 412Кб, 600x600
600x600
Эх, вот бы побывать на планете, где только один океан и посмотреть, что там происходит, какие организмы и вообще жизнь. Жаль, что никогда этого не узнаем.
Аноним 02/09/21 Чтв 15:19:40 677675303
>>677671
>Геостационар
>35880км
Ну подумаешь! Не 370км будет трос, а 35830км. Хуйня!
Аноним 02/09/21 Чтв 15:39:34 677679304
>>677675
Ты не пони маеш. Геостационар в смысле это говно сможет болтаться на малой высоте, но при этом не летая как ебанутое вокруг земли, а медленно (относительно поверхности) плавая, как говно в проруби небесной тверди.
Аноним 02/09/21 Чтв 15:50:07 677682305
>>677653
Так эта хуйня в 3.8 млрд световых лет от нас, а вот в радиусе 1000 световых лет от этой хуйни наверняка все разумные цивилизации выжгло, пиздец.
Аноним 02/09/21 Чтв 17:08:16 677697306
Аноним 02/09/21 Чтв 17:47:56 677703307
>>677682
> Так эта хуйня в 3.8 млрд световых лет от нас, а вот в радиусе 1000 световых лет от этой хуйни наверняка все разумные цивилизации выжгло, пиздец.
Так мы в 3.8 млрд световых лет оттуда. Как нас могло выжечь?
Аноним 02/09/21 Чтв 17:52:42 677704308
>>677673
> Эх, вот бы побывать на планете, где только один океан и посмотреть, что там происходит, какие организмы и вообще жизнь. Жаль, что никогда этого не узнаем.
Тебе не стремно? Вокруг тебя ничего, под тобой ничего. Не за что зацепиться и отдохнуть. Ты дрейфуешь и даже не можешь остановиться. Вокруг тебя буквально пустыня из воды, а в ее толще вполне возможно плавает что-то, своими щупальцами напоминающее кальмара. Стометрового. И нет, это не длина с щупальцами, а ширина мантии. А может быть и так что вода мертвая, и что в ней никого нет. Но это тоже пугает.
Аноним 02/09/21 Чтв 17:56:00 677705309
>>677675
Трос тяжеловатый будет только снизу, под космосом, а в космосе невесомый. Если сделать прочный трос, то все сработает.
Аноним 02/09/21 Чтв 18:00:34 677707310
>>677705
>под космосом
Ахаааахапхпхпавгдх!!
Аноним 02/09/21 Чтв 18:15:03 677711311
>>677703
ебанько, только не гори мне что ты умственно отсталый, который верит в исключительность человеческой цивилизации.
Аноним 02/09/21 Чтв 18:44:11 677715312
>>677704
Я же больше как помечтать, офк хули там делать, где одна вода. Просто интересно, что там на скорее всего пустой планете и вообще вся эта нетронутая природа (наверное она там есть). Короче я просто люто кайфую с этой хуйни как пустые планеты, которые только начинают свою жизнь.
Аноним 02/09/21 Чтв 19:03:07 677717313
>>677711
> ебанько, только не гори мне что ты умственно отсталый, который верит в исключительность человеческой цивилизации.
Пока не доказано обратное, не ебет что сказано.
Аноним 02/09/21 Чтв 19:07:08 677718314
>>677673
Такие планеты скорее всего будут безжизненны. и похожи на какой-нибудь похудевший Нептун.
Планеты с глобальным океаном как правило суперземли с низкой соленостью из-за фазового превращения воды на больших глубинах. Попутно отрезая подпитку минералами от вулканов.
В добавок такая планета будут покрыта густыми облаками, а атмосфера богата аммиаком и синильной кислотой. Свет от звезды не будет проходить до поверхности океана. Возможно на полюсах будут мощные ледники.
В глубоких океанах углекислый газ будет уходить вместе с минералами в клараты на дне.
Аноним 02/09/21 Чтв 19:18:38 677719315
>>677718
Очень круто. Мой мозг мечтателя очень доволен.
Аноним 02/09/21 Чтв 20:15:09 677726316
>>677719
Этот анон нафантазировал хуйни синильная кислота, что?!! но тебе похоже норм, приятного аппетита.
Аноним 02/09/21 Чтв 20:19:42 677727317
>>677726
Теперь твоя очередь, придумывай, что там на планете-океан.
Аноним 02/09/21 Чтв 20:25:32 677731318
Аноним 02/09/21 Чтв 20:32:57 677732319
Аноним 02/09/21 Чтв 20:34:53 677734320
>>672634
А есть такие же картинки, но для p- d- и f-орбиталей?
Аноним 02/09/21 Чтв 20:42:22 677736321
Чому после БВ не смогли синтезироваться тяжёлые элементы? Во взрывах сверхновых всяких могут, а тогда что мешало? ТЕмпературы ж поди и поболее бывали.
Аноним 02/09/21 Чтв 20:56:51 677740322
>>677726
>синильная кислота
HCN весьма распространена за снеговой линей, но там она вморожена в водяной или аммиачный лед.
В местах поближе к звезде, молекула уже разрушается от УФ излучения звезды. Однако молекула может прочно связываться с парами воды в облаках, тем самым спасаясь от фоторазложения. Да и сам океан будет служит хорошим резервуаром.
Аноним 02/09/21 Чтв 21:15:13 677743323
>>677736
Не существует стабильных или хотя бы долгоживущих нуклидов с массовыми числами 5 и 8. Поэтому синтез более тяжелых элементов требует особых условий вроде тройной гелиевой реакции.
И не все из возможных термоядерных реакций будет проходит. Это связано с изменением химического потенциала. Хоть реакции могут быть выгодны энергетически, но не термодинамически. Ну или по другому одна цепочка реакций будет блокировать другую.
Аноним 02/09/21 Чтв 21:23:08 677746324
Аноним 02/09/21 Чтв 21:26:27 677748325
>>677060
>Это кстати значит, что между 25 и 26 тоже лет 5-10 пройти может.

Напомнило анекдот:

Президент Туркменистана Сапармурад Ниязов в великой книге "Рухнама" с гордостью пишет, что туркмены изобрели колесо, письменность, выплавку металлов. Никто этого не отрицает. Просто другие народы в это время выпускали компьютеры и летали на Луну.
Аноним 02/09/21 Чтв 22:19:19 677756326
>>677743
>Не существует стабильных или хотя бы долгоживущих нуклидов с массовыми числами 5 и 8.
И тут (((они))) постарались?!
Аноним 02/09/21 Чтв 22:22:05 677757327
Аноним 03/09/21 Птн 00:51:04 677782328
20210903004926.jpg 217Кб, 962x864
962x864
>>677746
карл saga-н был хайпожорским пиздаболом, нихуя толкового не сделал, только пиарился без меры и тешил чсв. ты уверен что ты и есть этот беспоолезный кусок левацкого "астронома"? может ты чёткий Улугбек, запиливший обсерваторию из кизяков и камней?
Аноним 03/09/21 Птн 00:56:31 677783329
>>677740
ДА ШТО ТЫ СУКА НЕСЁШЬ? Нету никакой синильной кислоты в протопланетных дисках, есть соединение азота "аммиак" и соединение углерода "метан". Всё! Весь азот и углерод связаны в аммиаке и метане ещё есть свободный азот и сажа в камнях всякие там аминокислоты и синильная кислота в ничтожных количествах, опомнись! "Весьма распространена" у него, ишь ты - вода, вода весьма распространена. Синильная кислота нестойкое соединение, она разложится в солёном океане за пару миллионов лет.
Аноним 03/09/21 Птн 11:51:09 677831330
>>677736
Так атомов еще не было же. Пока частицы и античастицы устраивали разборки, вселенная расширялась, остывала. Когда разобрались, уже поздно было для тяжей, максимум водородину склеить смогли
Аноним 03/09/21 Птн 12:07:17 677833331
Гравиволны поперечные, так что если тела сближаются в горизонтальной плоскости, то вектор гравитации от волн будет колебаться в вертикальной? Схуяли?
Аноним 03/09/21 Птн 12:11:27 677834332
>>677833
нет никаких "гравиволн", есть только плотноприжатые сферы
Аноним 03/09/21 Птн 12:50:06 677836333
Аноним 03/09/21 Птн 14:42:39 677844334
>>677834
Поехавший, прими таблетки и пиздуй на хуй.
Аноним 03/09/21 Птн 16:34:16 677853335
Если космонавты на МКС начнут срать исключительно за борт, как быстро столкновение с замерзшей какахой станет статистически значимой угрозой для самой МКС и других аппаратов? Сможет ли какаха разъебать саму МКС, или там скорости будут незначительные, и все ограничится зашкваром?
Аноним 03/09/21 Птн 17:46:53 677860336
>>677853
Во-первых, какаха не замёрзнет - для свободно вращающегося чёрного тела на низкой земной орбите устанавливается температура около нуля цельсиев. Но вследствие вскипания воды её разорвёт на мелкие брызги ещё в момент высирания. После чего эти брызги весьма быстро высохнут, превратившись в сублимированный порошок, мелкие пылинки которого быстро затормозится в атмосфере и сгорят.
Аноним 03/09/21 Птн 18:58:00 677870337
fgergegergeg.JPG 103Кб, 1263x743
1263x743
>>672631 (OP)
Почему космический мусор не врезается в МКС?
Аноним 03/09/21 Птн 19:04:52 677872338
>>677870
Врезается. Постоянно.
От всего крупного МКС ловко уворачивается.
Плюс везение.
Аноним 03/09/21 Птн 19:18:15 677875339
>>677872
>От всего крупного МКС ловко уворачивается.
>Плюс везение.
Не стыдно?
Аноним 04/09/21 Суб 01:50:18 677925340
image.png 1934Кб, 1200x960
1200x960
image.png 180Кб, 440x250
440x250
image.png 3143Кб, 3008x2008
3008x2008
>>672631 (OP)
А вот у нас на планете доступный гелий заканчивается. Можем ли мы его в промышленных масштабах теоретически синтезировать?
Или проще с черпаком в космосе вылавливать солнечный ветер или шкрябать Сатурн?
Аноним 04/09/21 Суб 02:23:12 677931341
>>672631 (OP)
Есть ли древнеславянские названия у Марса и Венеры?
Аноним 04/09/21 Суб 03:09:39 677933342
Аноним 04/09/21 Суб 04:24:57 677948343
А когда узнали что звезды не просто серебряные гвоздики прибитые к небесному своду, а огненные шары вроде Солнца, в смысле не просто выдвинули предположение, а доказали с пруфами?
Аноним 04/09/21 Суб 05:28:05 677950344
uploads2Fcard2F[...].jpg 50Кб, 950x913
950x913
scale1200.jpg 166Кб, 1200x1200
1200x1200
Объясните, как сочетаются эти две официальные фотографии НАСА?
Аноним 04/09/21 Суб 05:31:12 677951345
>>677950
На обоих изображены Земля и Луна вместе. А что собственно не так?
Аноним 04/09/21 Суб 05:40:43 677952346
>>677951
Смотри на второй снимок и представь себя на поверхности луны. Будет земля такого же размера, как на первом снимке?
Аноним 04/09/21 Суб 05:45:01 677953347
>>677952
Первый снимок снимок сделан с орбиты Луны, поэтому на нем Луна большая, а Земля маленькая. Второй сделан с расстояния пару миллионов километров, поэтому на нем Луна маленькая, а Земля большая (ибо Земля больше Луны).
Аноним 04/09/21 Суб 05:46:03 677954348
uploads2Fcard2F[...].jpg 157Кб, 950x913
950x913
>>677951
Вот тебе, для наглядности? Как это так получается, кто объяснит?

кококо, фокус, выдержка, кудах
Аноним 04/09/21 Суб 05:49:08 677955349
>>677954
Объяснить что? Что обе Земли на твоем примере одинакового размера? Ну так ты сам совместил фото подогнав масштаб.
Аноним 04/09/21 Суб 05:52:13 677956350
>>677953
Т.е если сделать такой же снимок земли и солнца, где земля будет размером с монету, то солнце займет всю площадь снимка? По твоей логике?
Аноним 04/09/21 Суб 05:54:46 677957351
>>677953
Еще раз посмотри на второй снимок и представь, что ты приближаешься к луне. Луна будет приближаться и увеличиваться, а Земля будет уменьшаться. И в итоге у нас получится то, что на первом снимке. Так что-ли?
Аноним 04/09/21 Суб 05:55:56 677958352
220px-Transitof[...].jpg 2Кб, 220x170
220x170
>>677956
Внезапно да! вот тебе фото Венеры на фоне Солнца.
Аноним 04/09/21 Суб 05:57:54 677959353
>>677956
мне кажется он говорит про четкость левого снимка и отсутствие тени на Земле
Аноним 04/09/21 Суб 05:59:44 677960354
>>677958
А почему на этом снимке венера больше земли, лол? Что-то у вас там размеры как-то плавают, в космосе вашем.
Аноним 04/09/21 Суб 06:00:53 677961355
>>677957
Ну да, та и произойдет. Видимый размер Земли будет увеличиваться гораздо медленнее видимого размера Луны. Можешь провести эксперимент: посмотри в окно вытянув руку с выставленным большим пальцем, а потом медленно приближай руку к лицу, космические исследовакния в домашних условиях!
Аноним 04/09/21 Суб 06:02:16 677962356
>>677960
Но на этом снимке нет Земли, там только Венера и Солнце...
Аноним 04/09/21 Суб 06:06:38 677963357
>>677958
Т.е. если смотреть с Венеры, солнце будет относительно маленьким, потому что оно далеко, правильно? А когда ты ЕЩЕ ДАЛЬШЕ и от Солнца и от Венеры, то Солнце внезапно увеличивается. Правильно я понимаю?
Аноним 04/09/21 Суб 06:07:51 677964358
solar3.jpg 151Кб, 1000x561
1000x561
>>677962
Посмотри сравнительные размеры солнца, земли и венеры.
Аноним 04/09/21 Суб 06:11:50 677965359
>>677964
На твоей картинке Венера лежит прям рядышком с солнышком, в реальности между ними немалое расстояние, поэтому при фото с Земли Венера кажется больше.
Аноним 04/09/21 Суб 06:12:37 677966360
>>677961
Черт возьми, похоже, ты прав... Ладно, я еще вернусь.
Аноним 04/09/21 Суб 06:15:08 677967361
>>677959
>четкость левого снимка
Ты наверно имел в виду правого? В любом случае оба не старенькую нокию снимали
>отсутствие тени на Земле
Орбита Луны довольно-таки хитрозапутанная, поэтому падение тени Луны на Землю (сиречь солнечное затмение) явление довольно редкое.
Аноним 04/09/21 Суб 06:16:32 677968362
>>677966
>Ладно, я еще вернусь
Тут ты должен завернуться в плащ и зловеще захохотать.
Аноним 04/09/21 Суб 06:18:45 677969363
Аноним 04/09/21 Суб 07:05:56 677974364
Как же легко "закон перспективы" ебёт голову пориджа!
Аноним 04/09/21 Суб 08:35:29 677980365
>>677948
Анаксагор, 450 до н.э.
А, стоп, с пруфами? Примерно в середине 19 века, когда замерили дистанции и спектры.
Аноним 04/09/21 Суб 11:20:58 677992366
iss040e103941mi[...].jpg 817Кб, 4256x2832
4256x2832
>>677853
Смотри
- если ты просто насрёшь без импульса, какаха скорее всего сойдёт с орбиты, не задев МКС, но есть некая вероятность что через пару витков её пертубирует так что она шлёпнется о станцию обратно, так например чуть не случилось с сумкой инструментов, потерянной космочеловеком в 2008 при выходе в космос
- если ты насрёшь и выкинешь её в ретрогрейд ("назад"), она безопасно сойдёт
- если выкинуть её в прогрейд ("вперед"), она может шлёпнуться об станцию через виток
- самое опасное если выкинуть в нормаль ("вбок"), прилетит обратно уже через полвитка.

Собственно, по правилам эжекции хуйни в ВКД необходимо выкидывать именно в ретрогрейд. (например мелкоспутники запускали именно так)
А замёрзнуть какаха не замёрзнет, она закипит и высохнет быстро. Разъебать наверно не сможет, но погнуть или оторвать что-нибудь - вполне.

>>677870
Ещё как врезается, всякие неотслеживаемые болты десятками бахаются и повреждают всё что можно - радиаторы, панели, насосы, и даже теплощиты и иллюминаторы. Например в окна модуля Купол прилетало пару раз, поэтому сейчас держат баллистические шторки закрытыми от греха подальше, когда не смотрят в окна эти.
https://www.hou.usra.edu/meetings/orbitaldebris2019/orbital2019paper/pdf/6001.pdf
>The ISS impact database is maintained at the NASA/JSC by the Hypervelocity Impact Technology (HVIT) group. The database contains over 1,400 records of impact damage from ground-based observations of space-exposed hardware returned from ISS
>MMOD damage areas of interest acquired from EVA and IVA sources mentioned here are collected in an image database managed by the Image Science and Analysis Group at the Johnson Space Center (JSC). There are currently 380 records tagged as “MMOD” in the database, although it should be noted that some of these records include multiple MMOD damages (i.e., there are more than 380 MMOD impacts represented in this database).
Т.е. много сотен ударов, это только зафиксированных. Песчинки тупо никто не считает, нет возможности. Даже удар болтика в станцию (она огромная) определить это не самая тривиальная задача, для этого разрабатывают всякие йоба-микрофоны, нейросети для камер, датчики натяжения поверхности модулей, а пока обходятся визуальной инспекцией в основном.
Аноним 04/09/21 Суб 13:19:35 678054367
Стикер 63Кб, 512x415
512x415
Аноним 04/09/21 Суб 14:19:42 678084368
>>677925
Не заканчивается, это мем.
Синтезировать не можем.
Шкрябать Сатурн охуеешь.
Аноним 04/09/21 Суб 15:11:37 678106369
>>678084
>Не заканчивается, это мем.
Его запасы ограничены же, он не возобновляемый.

>Синтезировать не можем.
А как же альфа-частицы?
Аноним 04/09/21 Суб 15:17:44 678107370
>>678106
Ну так он заканчивается конечно - но примерно в том же смысле что нефть и газ. Ну и по факту как его добыча начнёт становиться дорогой, отпадут наиболее прожорливые потребители, ибо станет невыгодно.
>А как же альфа-частицы?
Не для промышленных масштабов.
Аноним 04/09/21 Суб 17:34:30 678157371
>>678107
А если будет водород-водородный синтез на каком-нибудь ДЕМО-2112, он же будет гелий в выхлопе иметь?
Аноним 04/09/21 Суб 21:58:36 678227372
>>677992
Всегда проигрываю с японского модуля, видевшего некоторое дерьмо
Аноним 04/09/21 Суб 22:04:21 678229373
>>677933
А теперь то же самое для космических тел
Аноним 04/09/21 Суб 22:30:00 678235374
>>677925
Сатурн, конечно! Это и будет основной продукт торговли колонии на Титане с Землёй. Вахтовики живут на Титане, по 4-5 месяцев работают на Энцеладе и Сатурне, добывают гелий и химические концентраты из ядра Энцелада. Гелием торгуют, концентраты (и сублимированные морепродукты) для внутреннего потребления. Для колонии из 1000 человек все потребности в белке будут закрыты одной поставкой с Энцелада в месяц с 6 тоннами сублимата. Ничтожная величина, учитывая что с Энцелада на Титан (с аэрозахватом) достаточно 4 км/с dV.
Аноним 05/09/21 Вск 16:57:35 678372375
>>678235
А разве до сатурна не годами нужно лететь? Вот круто, летел пять лет, пять месяцев поработал и обратно ещё пять лет лететь.
Аноним 05/09/21 Вск 18:11:58 678398376
>>678372
Лахтовики будут пять лет тралить на двачах.
Аноним 05/09/21 Вск 19:41:02 678436377
>>678398
С часовой задержкой тралить получится только в этом треде.
Аноним 05/09/21 Вск 21:23:01 678456378
>>678372
Пять лет туда, пять-десять там, пять обратно и на пенсию.
Движки с УИ как в экспансии реальны?
Аноним 05/09/21 Вск 21:43:07 678459379
Аноним 05/09/21 Вск 22:56:10 678468380
>>677961
Блин, вот всегда удивляюсь таким персонажам. Это же не какие то там черные дыры, которые своими глазами не разглядеть, для понимания которых надо скурить много матана. И то не поймешь всего.
С этими эффектами сталкиваются буквально ВСЕ в обычной жизни. Вот ты едешь в машине в сторону условного москва сити, вот тебя обгоняет суетолог на крузаке, вот только что крузак был огромным сараем рядом с тобой, вот теперь он на горизонте маленький на фоне высоких зданий. Ну ебана рот, ну как можно так тупить? Его наверное тоже рептилоиды сжимают в фотошопе в секретных лабораториях НАСА и транслируют прямо в моск.
Аноним 05/09/21 Вск 23:23:10 678473381
>>677705
>под космосом, а в космосе невесомый
это так не работает. Постоянно вижу это заблуждение в комментах к каким нибудь запускам, когда народ удивляется "а че это ракета летит не вверх а вверх и вбок".
Невесомость там от того что станция с космонавтами падает на землю постоянно, как невесомость в падающем лифте, если из шахты еще воздух откачать. Тебе уже сказали, космос это не как высоко, а как быстро. Если ты построишь 400 км башню до орбиты МКС, то поднявшись на нее ты обнаружишь, что земля тебя притягивает чуть чуть слабее, чем внизу и никакой невесомости нет и в помине. Сам себе задай вопрос, почему луна вокруг земли вертится. Со школы ты вспомнишь что тебе говорили "земля ее притягивает". Если в космосе невесомость в том понимании как видишь ее ты, то как земля ее притягивает, если вверху невесомость и все вокруг летает как хочет? Вспомни школу, гравитация бьет на бесконечность, ослабевая с квадратом расстояния, но даже другие звезды ничтожно, но притягиваются к земле, а земля к ним.
Чтобы стать как мкс, ты должен залезть в пушку на этой башне и выстрелить себя параллельно земле с первой космической скоростью. Вот тогда ты будешь падать на землю, а земля под тобой заворачиваться, так как она круглая. бинго, ты на орбите в невесомости. Замедлишься, упадешь и очень больно.
Аноним 06/09/21 Пнд 00:46:55 678487382
>>678372
Лететь с колонии на Титане, а не с Земли разумеется. Титанцы - новый народ, свободное государство, им всякие хуи с грязной вонючей Старой Земли с леваками, квотовиками и метисами-вырожденцами не нужны. Титан это фронтир, там скорее всего будут устойчивые психикой первопроходцы с ценностями Среднего Запада. Так что "вахтовики" это такие жители Титана, просто кто-то работает на самом Титане (строит города, выращивает урожай, а кто-то на Энцеладе концентраты добывает и разводит рыбу). Надо кстати придумать, как бы на Энцеладе обустроить пищевую цепь на хемосинтетических бактериях. А то от самого Энцелада тогда мало толку, без использования местных ресурсов: рыбу в бассейнах и на Титане растить можно.
Аноним 06/09/21 Пнд 12:41:32 678568383
Если мы на Ганимеде обнаружим форму жизни, например метановую водоросль. Как мы будем называть эту жизнь?
Ганимединская? Ганимедская? Ганимедианская? Ганимедийская?
Аноним 06/09/21 Пнд 12:56:24 678570384
>>678568
На Ганимеде никакой мы жизни не обнаружим, это заведомо безжизненный кусок грязи. На Европе может быть, жизнь будет европеанская. На Энцеладе - энцеладианская. На Титане - титанианская, на Титании - титаниянская.
Аноним 06/09/21 Пнд 13:08:28 678574385
image 87Кб, 500x500
500x500
Аноним 06/09/21 Пнд 13:12:27 678575386
image 305Кб, 985x867
985x867
Аноним 06/09/21 Пнд 15:18:22 678597387
image.png 97Кб, 593x636
593x636
image.png 16Кб, 550x251
550x251
>>678575
.post {min-width: 1000px !important}
Аноним 06/09/21 Пнд 15:38:38 678598388
Какого размера нужно развернуть зеркало/фольгу на Луне, чтобы их блеск можно было увидеть с Земли через любительский телескоп или даже бинокль?
Аноним 06/09/21 Пнд 16:50:23 678609389
>>678598
Около 3 км будет видно в телескопы типа 114 мм рефлектора гарантировано, как яркую звездочку на поверхности. Может и сильно меньше можно
Аноним 06/09/21 Пнд 16:56:44 678610390
image 36Кб, 457x454
457x454
image 88Кб, 500x1125
500x1125
>>678597
Охуенно, спасибо.
Аноним 06/09/21 Пнд 16:57:18 678611391
А ёбнувший в завров метеорит испарился или утонул в мантии или под ногами практически валяется всё ещё?
Аноним 06/09/21 Пнд 17:03:03 678612392

>>678609
А чтобы без телескопа и бинокля. А просто глазами увидеть это зеркало/фольгу?
Аноним 06/09/21 Пнд 17:15:35 678613393
5Q8M6Mo.png 86Кб, 938x348
938x348
>>677652
могнитары и посильней пики ебошут
Аноним 06/09/21 Пнд 17:37:07 678619394
>>678574
Жизнь-в-землянке
Аноним 06/09/21 Пнд 20:13:28 678649395
Стикер 63Кб, 512x411
512x411
>>678613
>В гамма-диапазоне взрыв был ярче полной луны
Ну дык ёпт, это несложно, луна в гамме считай нихуя что не светит.
Аноним 07/09/21 Втр 06:32:51 678689396
Аноним 07/09/21 Втр 08:57:48 678696397
Аноним 07/09/21 Втр 09:00:41 678697398
карасик
Аноним 07/09/21 Втр 12:35:08 678709399
Nemesis&Qarasiq[...].jpg 35Кб, 512x384
512x384
20210907123312.jpg 1182Кб, 1850x3290
1850x3290
>>678697
Карасик - спутник Немезиды, как Эрида и Дисномия
Аноним 07/09/21 Втр 14:37:51 678719400
1631014668038.mp4 1499Кб, 720x720, 00:00:13
720x720
В чем космический эффект располагать солнечные панели на холмах?
Да ещё и со всех сторон (!).
Неэффективно же.
Ладно если бы они были расположены перпендикулярно небесной дуге солнца под средневыгодным углом.
Так нет блять, тупо отпизды налепили со всех сторон
Аноним 07/09/21 Втр 15:19:22 678723401
>>678719
Как и в случае распаханных вдоль и поперёк гор в иных местах - занимают непригодные для остального площади, очевидно же. Панели стоят как грязь, а лепездричество дают.
Аноним 07/09/21 Втр 15:21:27 678724402
>>678689
В чём тупой вопрос? А вообще что-то ты поздно, ещё и с мурзилками. >>676765 → До сих пор их гипотеза - самая слабая, но самая раскрученная.
Аноним 07/09/21 Втр 15:46:42 678729403
>>678719
Но их не показали со всех сторон. Полагаю, их показали только с солнечной стороны.
Аноним 07/09/21 Втр 21:34:26 678821404
image.png 2054Кб, 1200x675
1200x675
Тред про навозных жуков проебан?
Аноним 07/09/21 Втр 21:38:00 678822405
>>678821
Ты ещё про безносые рисунки спроси.
Аноним 07/09/21 Втр 22:00:37 678839406
>>678821
его судьбу нельзя обсуждать кое-кто в припадке удалил его, антропоморфные изображения плонет и перекат картинкотреда
Аноним 07/09/21 Втр 22:00:47 678840407
>>678821
Да. Уже с годик-другой как был вероломно потёрт
Аноним 07/09/21 Втр 22:13:35 678846408
>>678840
даже года не прошло, где-то между октябрём и декабрём ещё катился до сих пор в кеше ведра висит, будет что показать потомкам
Аноним 08/09/21 Срд 01:54:53 678881409
Можно ли взлететь на ракете сконструированной на говне?

Насколько это целесообразно?
Аноним 08/09/21 Срд 05:09:11 678885410
>>678881
Можно взлететь чуть ли не на всём что горит. Говно высушить, измельчить в пыль, перемешать с окислителем, стабилизатором, пластификатором, сформовать шашки. Получится ТТРД с весьма хуёвым импульсом, и насчёт стабильности горения тоже сомневаюсь. Насрать придётся овердохуя, и как-то контролировать состав говна. (жрать что-то одно, например)
Аноним 08/09/21 Срд 05:12:35 678886411
>>678881
>>678885
Достаточно заложить говно в биореактор вместе с конструктором и получить метан, который затем очистить и сжижить. Охуенное топливо, хоть старшип заправляй.
Аноним 08/09/21 Срд 06:00:35 678887412
ракета на конно[...].png 49Кб, 1238x528
1238x528
>>678881
В одном из предыдущих тредов ракету на конной тяге обсуждали, с лошадиным говном в качестве рабочего тела.
Аноним 08/09/21 Срд 08:04:05 678894413
Пишут, эпоха рекомбинации, когда вселенная остывала до прозрачности, длилась 100000лет. Но был ли толк в этой прозрачности, можно ли было бы хоть что-нибудь глазом различить во всей этой печке вокруг? Какую-нибудь рябь из облаков/волокон из не успевшей ещё охладиться плазмы и вновь нагревающейся, например. Или всё было однотонным?
Аноним 08/09/21 Срд 08:36:05 678896414
1.Квантовый вакуум, он же пена из виртуальных частиц, возникающих и исчезающих,- это обязательно пары частиц-античастиц или могут целые цепочки превращений происходить (произвольной длины)? И массы их могут быть вплоть до максимона, а время жизни вплоть до планковского а то и за пределом? Могут там прятаться монстры в в тысячи-триллионы гэв массой?
2. Считают ли сейчас, что всяких частиц может быть на порядки больше или что почти все уже открыты?
Аноним 08/09/21 Срд 08:36:35 678897415
Аноним 08/09/21 Срд 11:07:05 678916416
image 96Кб, 500x500
500x500
>>678887
Оригинальное на самом деле решение для частичного отбора химической энергии рабочего тела на привод подачи сена и говновентилятор.
Аноним 08/09/21 Срд 11:14:26 678917417
>>678896
>это обязательно пары частиц-античастиц
Обязательно пары.
>>678894
Тогда повсюду, во всем пространстве было сплошное горячее плотное вещество (хотя плазмой это уже не было, ведь электроны объединились с ядрами в атомы), которое постепенно остывало, да, и комковалось из-за неоднородностей плотности, из которых потом выросли звезды, галактики, скопления галактик и, наконец, крупномасштабная структура. В принципе, если бы ты мог наблюдать за этим миллиарды лет, то увидел бы формирование всего этого.
Аноним 08/09/21 Срд 11:51:35 678923418
>>678917
>если бы ты мог наблюдать за этим миллиарды лет, то увидел бы формирование всего этого
Увидел бы не больше того, что сейчас видно на границе наблюдаемой вселенной и чуть ближе.
Аноним 08/09/21 Срд 16:01:01 678952419
Если в чд пихнуть неразрушимый шар, то при пересечении гс он получит бесконечную энергию и чд расширится до всей вселенной и всех нас убьёт совсем?
Аноним 08/09/21 Срд 16:06:55 678953420
>>678917
НЕ, вика пишет, рекомбинация 100к лет длилась, так я и спрашиваю, был ли этот процесс сколь-нибудь зрелищным
>>678917
Аноним 08/09/21 Срд 16:26:01 678955421
Аноним 08/09/21 Срд 16:53:12 678959422
Ну и чем это не религиозная догматика?

>>678894
>>678896
>>678917
>>678952
>>678953

>Ну в общем хуй знает что было, может так было
>А вот ещё вот так может нахуй было, ну хуй знает
>Магический дух в общем
>Ууу сила ноуки
Аноним 08/09/21 Срд 17:32:38 678962423
>>678952
Дело за малым - найти неразрушимый шар.
Аноним 08/09/21 Срд 17:48:15 678966424
>>678962
> Дело за малым - найти неразрушимый шар.
Его можно слепить из ниггеров
Аноним 08/09/21 Срд 18:26:29 678970425
>>678952
Нет, потому что никаких, блядь, неразрушимых шаров не существует в природе, а при попадании в черную дыру все безвозвратно падает в центр и там разрушается к хуям, навеки исчезая в сингулярности.
Аноним 08/09/21 Срд 18:27:59 678971426
??? ????????.jpeg 3Кб, 200x149
200x149
>>678959
>Ну и чем это не религиозная догматика?
Аноним 08/09/21 Срд 18:33:49 678972427
>>678970
А если на черную дыру уронить сингулярность че будет?
Аноним 08/09/21 Срд 19:15:42 678977428
>>678972
Голых сингулярностей не бывает, принцип космической цензуры запрещает. Можно уронить черную дыру на черную дыру. Будет слияние черных дыр с испусканием гравитационных волн, что уже детектили множество раз.
Аноним 08/09/21 Срд 19:27:32 678980429
image.png 55Кб, 1665x724
1665x724
>>678977
А если не закруглять орбиты ЧД как щас обычно они, а по касательной ебнуть или по прямой, че будет?
Аноним 08/09/21 Срд 19:55:59 678982430
>>678980
будет то же самое, только меньше волн вплоть до одной
Аноним 08/09/21 Срд 19:59:15 678983431
>>678980
То же самое будет.
Аноним 08/09/21 Срд 20:32:03 678991432
>>678980
будет очень мало гравитационных волн. если дыры летят лоб в лоб то вообще будет один импульс, притом по мощности самый обычный
Аноним 08/09/21 Срд 22:20:58 679003433
jameswebb.jpg 106Кб, 940x528
940x528
А вот если телескоп Джеймс Уэбб бабахнет вместе с ракетой во время запуска, что тогда, ммм???
Аноним 08/09/21 Срд 23:49:42 679009434
>>679003
Пизда тогда. Годы трудов и миллиарды бачей пойдут в жопу ниггера.
Аноним 09/09/21 Чтв 01:18:42 679018435
Аноним 09/09/21 Чтв 01:58:29 679021436
image.png 1479Кб, 1000x668
1000x668
Я тут задумался, а че, вся вселенная скорее всего это скорее как сфера-облако с неровными краями на полотне бесконечной пустоты? В том плане, что откуда ты с земли на ракете не влети низ будет низом, а верх верхом. Газуй куда хочешь и даже имей ты при себе бесконечные световые года времени, чтобы добраться так сказать до конца вселенной, то быть может дойти до "конца карты", то скорее всего ты вывалишься в реальную пустоту или сама по себе вселенная и есть уже эта бесконечная пустота без края? То если пофантазировать и рассматривать с микроскопа всю эту вселенную, то какой формы она будет?

Аноним 09/09/21 Чтв 02:21:21 679022437
>>679021
это если совсем плоская, вообще предпологается некая глобальная минимальная кривизна дающая околобесконечную 3-сферу и условный полёт в одном направлении со скоростью выше расширения вселенной может привести обратно в ту же условную точку
Аноним 09/09/21 Чтв 12:15:59 679110438
>>679021
Вселенная - замкнутое трехмерное пространство, трехмерная гиперсфера. У нее нет краев по определению. Если бы она не расширялась, ты бы мог совершить кругосветное путешествие по ней - вылететь из любой точки и через какое-то время, просто летя по прямой, вернуться обратно в эту же точку.
Аноним 09/09/21 Чтв 12:44:35 679120439
>>679110
Это охуенно. Еще мне нравится, что для вселенной нет понятия начала. В человечком понятии казалось бы у всего должно быть начало и конец, как отрезок времени. А вселенная существовала всегда. И ты типа думаешь, ну бля может она хотя бы миллиарды лет назад сперва появилась пустой, но нет нихуя. Просто всегда была, никто её не создавал, она просто была как самая великая аномалия. Просто была, без начала.
Аноним 09/09/21 Чтв 12:46:16 679121440
image.png 2721Кб, 1820x1137
1820x1137
Аноним 09/09/21 Чтв 13:12:04 679123441
>>679120
Экстраполируя наблюдаемое расширение Вселенной при использовании решений общей теории относительности назад во времени, мы за конечное время, равное возрасту Вселенной - 14 млрд лет, придем к состоянию Вселенной, характеризующемуся бесконечной кривизной пространства-времени, - сингулярности. Так что у Вселенной безусловно есть начало. Это основа современной космологии.
Аноним 09/09/21 Чтв 15:50:06 679168442
>>679121
>>679123
Ладно, проебался с теорией взрыва. Но нельзя ли предположить, что до этого взрыва уже была какая-то почва пустоты, которую никто не создавал и вот она была без начала, просто всегда существовала. Я понимаю, что почти все при взаимодействии с чем-то дает определенную реакцию, что по сути процесс от начала и конца. Но может та пустота существовала сама по себе и не нуждалась в создании.
Аноним 09/09/21 Чтв 16:00:24 679172443
>>679168
Не было никакой пустоты. Не было самого момента "до" появления Вселенной. Само пространство-время появилось, когда появилась Вселенная ведь Вселенная - это и есть пространство-время. Сингулярность - это состояние, при котором кривизна пространства-времени является бесконечной. Бесконечная кривизна означает, что число измерений пространства-времени равно нулю, размер Вселенной равен нулю. Это означает, что у тебя просто нету ничего, сингулярность - подлинное ничто. Можно дать весьма логичное и правдоподобное квантово-механическое описание туннелирования этого ничего в маленькую замкнутую вселенную. Т.е. это спонтанное квантовое рождение вселенной из ничего. Скорее всего, наша Вселенная так и появилась.
Аноним 09/09/21 Чтв 16:03:48 679174444
>>679172
>спонтанное квантовое рождение вселенной из хуй знает чего
Поправочка. Известная физика не даёт ответа, было ли там ничего или чего. Там было хуй знает что.
Аноним 09/09/21 Чтв 16:23:04 679177445
>>679172
А почему тогда произошло пробитие бесконечной кривизны сингулярности
Аноним 09/09/21 Чтв 16:57:57 679182446
>>679174
ОТО приводит к сингулярности. Сингулярность - это ничто, отсутствие пространства-времени. Современная космология основывается на ОТО. Так что современная физика прямо говорит, что "там" было ничто. Если точнее, то никакого "там" не существовало, т.к. не было самого пространства-времени.
>>679177
Это квантовый процесс, а потому вопрос "почему" тут не имеет смысла. Квантовые процессы происходят сами собой, без причины, спонтанно.
Аноним 09/09/21 Чтв 18:01:20 679193447
1.png 1Кб, 291x96
291x96
Допустим люди добывают минералы из крупного астероида в поясе астероидов и уже извлекли 10% его массы. Изменится ли орбита астероида вокруг Солнца из-за уменьшения массы и соответственно силы гравитационного притяжения? Или вообще ничего не изменится?
Аноним 09/09/21 Чтв 18:07:54 679195448
>>679193
Ничего не изменится.
Аноним 09/09/21 Чтв 19:15:29 679212449
>>679193
Если просто изъять массу и увести ее, не трогая остальное, то ничего не измениться.
Однако это практически не реализуемо.
Разработка минералов на астероиде требует источников энергии и самый простой вариант это использование света Солнца. Для выхода про промышленную мощность, придется использовать громадную собирающую площадь, которая будет работать как солнечный парус. Сюда же добавляется тяга от сброса мусорного тепла. Суммарная тяга в итоге будет шатать орбиту. Для больших астероидов это не актуально, а вот для маленьких это уже значительно.
Другой фактор это солнечный ветер. Он становится главным загрязняющим фактором в производстве. Конечно его можно отклонить магнитным полем, но это отклонение будет создать непредсказуемую тягу, пидарасищую орбиту.
Это была мизерная хуита, актуальная при долгоиграющих миссиях. Поскольку основное вещество это оксиды и углистое говно, то побочные продукты будут кислород и углерод, которые надо куда-то девать. Углерод еще можно складировать, а вот с кислородом проблема, поскольку он газ и при испарение будет создать значительную реактивную тягу. И один сброс большего количество газообразного кислорода может даже изменить орбиту большего астероида.
Аноним 09/09/21 Чтв 23:46:40 679254450
image 243Кб, 1289x1235
1289x1235
>>679177
С отрицательного измерения постучали.
Аноним 10/09/21 Птн 01:35:56 679265451
>>679193
Если пулять добытой рудой из рельсовой пушки, то да, изменится.
Аноним 10/09/21 Птн 05:17:53 679294452
Как строить жилую станцию на десятки тысяч человек в космосе, если она будет дырявиться как МКС?
Аноним 10/09/21 Птн 08:36:16 679303453
>>679294
Как и МКС - модульной. Модуль отсоединил, утопил, присоединил новый.
Аноним 10/09/21 Птн 08:50:46 679304454
>>679182
>ОТО приводит к сингулярности. Сингулярность - это ничто, отсутствие пространства-времени.
Даже точечная сингулярность это "хуй знает что", ибо всё это работает максимум до планка. А есть ещё кольцевые сингулярности в ОТО, например во вращающихся ЧД, то есть вообще считай в любых - это уже 2Д структура с ненулевыми размерами. (конкретно у Вселенной момента вращения не обнаружено, во всяком случае пока)
Ну и "отсутствие пространства-времени" это очень условное понятие, при том что "перешагнуть" за пределы сингулярности модель не может ни в том, ни в другом направлении. Это модель с пространством-временем ломается на этой границе, а не реальность. ТО как известно неполна и не является моделью всего.
Аноним 10/09/21 Птн 09:18:03 679306455
>>679304
Кольцеобразная сингулярность скорее 1Д, т.к. у неё предполагается всего один размер — радиус кольца. Ни "ширины", ни "высоты" у неё нет.
Аноним 10/09/21 Птн 11:24:20 679313456
>>679304
Сингулярность в ОТО - это состояние с бесконечной кривизной пространства-времени. Бесконечная кривизна означает, что число измерений пространства-времени равно нулю, размер вселенной (если в таком состоянии находится вся вселенная, как это было с нашей Вселенной в самом начале) равен нулю. Т.е. самого пространства-времени в таких условиях у тебя не существует. Поэтому это именно ничто, а ты несешь хуйню.
>то есть вообще считай в любых
Большинство черных дыр вращаются достаточно медленно, поэтому для их описания применяют простейшую модель шварцильдовской черной дыры с точечной сингулярностью в центре. Решение Керра с кольцевой сингулярностью применяется лишь в случаях предельного вращения черной дыры, что бывает не так часто. Так что ты снова спизданул хуйню.
Аноним 10/09/21 Птн 11:35:05 679314457
>>679313
>Т.е. самого пространства-времени в таких условиях у тебя не существует.
Ты похоже вообще нихуя не понял на что отвечаешь
Аноним 10/09/21 Птн 12:00:19 679319458
Аноним 10/09/21 Птн 14:32:51 679335459
>>679003
Это лишь одна проблема и то маловероятна, куда хуже когда телескоп на L2 притащиться и при раскрытии и диагностики первой работы что то пойдет не так, кто к нему полетит?, люди вроде как дальше луны то не летали
Аноним 10/09/21 Птн 14:34:33 679336460
1) Как с помощью математики узнать своё положение на орбите неизвестной планеты (ну там, полюс, экватор, умеренные широты)? Можно ли вычислить орбиту планеты вокруг местной звезды?

2) Экваториальный радиус несколько больше полярного, ну и гравитация чуть сильнее на экваторе. Значит, самые сильные гравитационные пики должны быть у буквальных поверхностных пиков, то есть гор? У Эвереста самая сильная гравитация, получается? Почему тогда вокруг Эвереста нет куполообразного вздутия атмосферы?
Аноним 10/09/21 Птн 14:36:00 679338461
>>679335
Никто не полетит, будут тилибонькать софтом.
Аноним 10/09/21 Птн 14:46:23 679341462
>>679336
>Как с помощью математики узнать своё положение на орбите неизвестной планеты
По движению звезд ночью, широту легко определить, но для долготы нужна опорная точка на планете и точные часы.
>Можно ли вычислить орбиту планеты вокруг местной звезды?
Через движение других планет звезды и необязательно, но это все упрощает, спутник планеты.

Гравитация на экваторе наоборот слабее. Поскольку он дальше удален от цента масс.
Поверхность Земли относительно гладкая и практически не влияет на гравитацию. Больше влияет неоднородности распределения плотности в мантии.
Аноним 10/09/21 Птн 14:57:44 679345463
Через десятилетий или веков люди смогут прокопать дыру в земле глубже 12км? Там же уже сложность не сколько в глубине, сколько в температуре, которая достигает 200 градусов. Эх, интересно, что там за камни в глубине, потому что уже на этой глубине золото находили.
Аноним 10/09/21 Птн 15:16:19 679347464
>>679345
Там алмазы и лавовые бассейны, а дальше уже бедрок, его не прокопать.
Аноним 10/09/21 Птн 15:21:37 679348465
>>679347
А еще если в лаву уронить куклу вуду, то стена плоти появится, небось?
Аноним 10/09/21 Птн 15:23:07 679349466
image 149Кб, 1404x824
1404x824
>>677734
Бамп тупому вопросу.
Аноним 10/09/21 Птн 15:29:07 679350467
>>679341
>По движению звезд ночью, широту легко определить, но для долготы нужна опорная точка на планете и точные часы.
Как?
>Через движение других планет звезды и необязательно, но это все упрощает, спутник планеты.
Как?
Аноним 10/09/21 Птн 16:01:54 679351468
800px-StarTrail[...].jpg 153Кб, 800x1200
800x1200
>>679350
Широту определяем через угол наклона к горизонту небесного полюса - точки вокруг которого вращаются звезды за ночь.
Долгота это отклонение местного времени от опорного. Местное время измеряется по солнцу или звездам. Опорное время получают от часов, которые синхронизированы с временем с опорной точки.

Планеты двигаются относительно далеких звезд и солнца, собрав достаточно наблюдательных данных и приняв, что они двигаются по закон Кеплера, вычисляем параметры их орбиты, из получаем данные о движение солнца,а из него и нашу орбиту.
Если есть затмевающих спутник, от все упрощается, мы получаем прямые данные и положении солнца.
Аноним 10/09/21 Птн 16:04:40 679352469
>>679349
>Бамп
Ответом?
Вот эти вот картинки с намотанными волнами де Бройля - просто-напросто модель для облегчения расчётов была. На самом деле ведь электроны не летают ни по круговым орбитам ни по кривым.
Аноним 10/09/21 Птн 16:15:45 679353470
>>679352
>Ответом?
Ну, блин, хотелось бы визуализацию в 3d этих типа как волн, размазанных по более сложной фигуре, чем окружность. Наглядная же интуитивно красивая модель была бы. По мне так и Боровская модель не теряет охуенности.
https://www.youtube.com/watch?v=67gbGCJjR_E
Аноним 10/09/21 Птн 16:18:53 679354471
Аноним 10/09/21 Птн 16:24:51 679355472
>>679354
Это видел, спасибо, крутая вещь, но там просто облака вероятностей, без попытки в изображение стоячей волны на поверхности.
Аноним 10/09/21 Птн 16:26:08 679356473
>>679355
Ну там и есть визуализация волновой функции. Цвет фаза, яркость амплитуда.
Аноним 10/09/21 Птн 20:23:46 679400474
Когда уже технологии дойдут до того что бы рассматривать поверхность экзопланет на таком же уровне как Луну с Земли через бинокль?
Аноним 10/09/21 Птн 21:24:45 679409475
Аноним 11/09/21 Суб 05:39:27 679429476
>>679400
> Когда уже технологии дойдут до того что бы рассматривать поверхность экзопланет на таком же уровне как Луну с Земли через бинокль?
Никогда в спейс инжин не играл?
Аноним 11/09/21 Суб 05:42:51 679430477
Господа, давненько читал небольшой рассказик про дрейфующий в космосе поврежденный корабль, они находились тысячи лет в космосе, процветал каннибализм и все в духе, в конце их нашли и охуели, подскажите название пж
Аноним 11/09/21 Суб 06:39:23 679433478
>>679430
Пасынки Вселенной
Аноним 11/09/21 Суб 07:03:23 679434479
>>679400
Тысячекилометровый телескоп мне построй, быстроблять
Аноним 11/09/21 Суб 13:23:50 679447480
>>672631 (OP)
Поясните за время Ляпунова. Я правильно понимаю, что на масштабах в десятки-сотни миллионов лет, как в прошлом так и в будущем орбиты планет в солнечной систем непредсказуемы?
Аноним 11/09/21 Суб 13:35:52 679448481
>>679433
>в конце их нашли
В Пасвинках Вселенной другая концовка. Но сюжет похожий.
Аноним 11/09/21 Суб 13:46:04 679449482
Аноним 11/09/21 Суб 16:49:31 679454483
Что если, в условиях невозможности подтвердить/опровергнуть одинаковость скорости света между двумя объектами (в прямом и обратном направлении), вообще нахуй отказаться от этого постулата за ненадобностью? Может ли наличие такой разности скоростей света объяснять/быть причиной всяческих вселенских артефаков, вроде различий теоретической и наблюдаемой скоростей вращения элементов галактики, выдумывания тёмной материи и прочего словоблудия?

Скажем, свет (да и сама гравитация заодно, чего уж там) медленнее распространяется вдоль отрицательного градиента гравитации, при движении от центра галактики, и быстрее распространяется вдоль положительного, к центру, но это не засечь простым измерением разности между временем излучения и временем приёма отражённого с другой стороны сигнала, т. к. в среднем всегда будет получаться c.

https://www.youtube.com/watch?v=pTn6Ewhb27k
Аноним 11/09/21 Суб 17:00:10 679455484
>>679454
Самое худшее видео Веритасия. В отличие от других его видео это - полнейший идиотизм, я не смог до конца досмотреть.
Аноним 11/09/21 Суб 17:04:21 679456485
>>679455
Почему? Довольно-таки логичные выводы из отказа от Эйнштейновского постулата одинаковости скорости при измерениях с использованием зеркала. Никто же не утверждает, что оно на самом деле так, просто попытка выкинуть одну из аксиом, как было когда-то с непересечением параллельных прямых, например.
Аноним 11/09/21 Суб 17:11:46 679457486
>>679454
Ты хуйню спизданул, шизик. Таблетки прими.
Аноним 11/09/21 Суб 17:13:38 679458487
>>679455
>>679457
Попытайся непротиворечиво оформить свою мысль, пожалуйста. Или же иди на хуй, быдло.
Аноним 11/09/21 Суб 17:34:03 679463488
>>679449
Тогда почему этот факт никак упоминается в истории земли? Ведь это же элементарный и очевидный ответ на вопросы почему в каменоугольный период было так жарко, почему случилась и почему закончилась земля-снежок и тд.
Аноним 11/09/21 Суб 17:40:02 679466489
>>679463
Возможно, потому что этот ответ ни на что не отвечает, а просто констатирует, что «ну вот так вот, хуй знает короч».
Аноним 11/09/21 Суб 17:51:29 679469490
>>679433
Это был не роман, а небольшая притча на пару страниц, там люди жившие в темноте веками, ослепшие и озверешвие, помню только что их нашли.
Аноним 11/09/21 Суб 18:21:52 679472491
>>679458
На хуй твоя жопа хороша, чушок.
Аноним 11/09/21 Суб 22:36:09 679506492
>>679454
В спейсаче это уже разбирали.

https://2ch.hk/spc/res/635057.html#637183

Эту гипотезу можно опровергнуть буквально за 1 минуту двумя дырками в картонке и лазерной указкой. При переменной скорости света картина интерференции зависела бы от угла поворота установки, т.к. свет проходит два пути за разное время и фаза излучения перестаёт совпадать.
Аноним 11/09/21 Суб 23:38:33 679508493
>>679506
Я вообще не понимаю, что сподвигло его записать такой безграмотный видос. Он в основном годные видео снимает, этот Дерек Миллер (или как там его), и физическое образование у него есть. Но на этом видосе что-то пошло не так. Как можно было так объебаться с такой элементарной вещью вопрос риторический, если что?
Аноним 12/09/21 Вск 00:15:00 679515494
>>679463
>в каменоугольный период было так жарко
Ты такскозал?
Аноним 12/09/21 Вск 01:11:30 679519495
image 286Кб, 1021x535
1021x535
>>679506
О, это выглядит похоже на разъёб по фактам. Я так понимаю, если бы скорость зависела от направления, то при повороте всей установки на экране появлялась бы асимметрия картины, аналогичная той, которая происходит при повороте экрана установки относительно источника излучения (но эта асимметрия будет гораздо менее выражена, за счёт некоего коэффициента, зависящего от разности скоростей для данных направлений в данной точке пространства). Вот единственное, что меня теперь останавливает от согласия, так это то, что я не знаю, проверялось ли это экспериментально на больших по размеру установках с точными измерениями. Дома с дырявой картонкой это скорее всего не обнаружить, а всякие LIGO этому эффекту вообще не должны быть подвержены (или должны? хуй знает, уже не уверен), ибо там как раз-таки применяются зеркала, сводящие на нет различие скоростей туда/обратно. В любом случае спасибо за направление мыслей в нужном (надеюсь) направлении.
Аноним 12/09/21 Вск 09:37:57 679539496
1344498303527.jpg 41Кб, 607x720
607x720
Как и в какой среде и при каких условиях можно сжечь кремнийорганическую "древесину"?
Аноним 13/09/21 Пнд 04:34:39 679667497
image 39Кб, 921x417
921x417
Как должна выглядеть гравитационная линза для гравитационного телескопа, чтоб она не распидорашивала объект наблюдения на 4, а фокусировала? Я так понимаю, для впуклой линзы нужен массивный тор. А что взять в качестве выпуклой, чтобы она умела в фокусировку?
Аноним 13/09/21 Пнд 05:17:43 679668498
>>679667
Голова у тебя выпуклая, дурачок.
Аноним 13/09/21 Пнд 06:49:18 679671499
>>679668
>>679472
>>679457
Нет, это с тобой что-то не так. Зачем-то пытаешься в треде тупых вопросов выглядеть умнее, высираясь тупыми ответами.
Аноним 13/09/21 Пнд 10:05:04 679685500
>>679539
В газообразном франции под давлением в 200 атмосфер горит всё. А что не горит, то взрывается.
Аноним 13/09/21 Пнд 10:19:33 679687501
>>679685
>горит
>в щелочном металле, который в общем-то наоборот сильный восстановитель
И каким образом оно это делает? Ладно бы фтор предложил.
Аноним 13/09/21 Пнд 11:22:47 679691502
Когда уже будут нормальные двигатели?
Доколе будем дрочить на гигантские бочки с керосином, которой не хватает даже на первую космическую, что приходится втыкать по 3 ступени.
Аноним 13/09/21 Пнд 11:24:59 679693503
>>679671
Дегенерат, это >>679668 другой анон, если что. Завязывай уже с боярой, ты и так тупой.
Аноним 13/09/21 Пнд 11:33:25 679694504
>>679693
Если два долбоёба ведут себя одинаково, выглядят одинаково и воняют на весь тред одинаково, какая разница, другой или не другой? Вы оба — рак в этом треде.
Аноним 13/09/21 Пнд 11:55:11 679700505
>>679691
>Когда уже будут нормальные двигатели?
Лет через 20 в лучшем случае, если
1. Металлический водород метастабилен и его можно хранить.
2. Его получится получать в промышленных масштабах.
3. Его получится удобно использовать как топливо.
Кроме него единственный двигатель который может быть мощнее окисления топлива это ядерный, но никто не будет его в атмосфере использовать это точно.
Аноним 13/09/21 Пнд 11:56:16 679701506
>>679694
Не плачь, пидорок.
Аноним 13/09/21 Пнд 12:11:41 679704507
>>679700
>Металлический водород
А окислять его чем? Металлическим кислородом? Один хуй морозилка нужна будет.
Аноним 13/09/21 Пнд 12:24:57 679709508
Аноним 13/09/21 Пнд 12:33:06 679711509
>>679704
У жидкого кислорода плотность повыше чем воды, не такая уж проблема.
Вся проблема с хранение водорода. Компактно хранить его можно только при высоком давлении. Но особый интерес представляет его металлическое состояние, при котором с ним можно делать различные полезные манипуляции, снимающие издержки от емкости высокого давления. А совсем мокрая писечка это сверхпроводящий водород.
Аноним 13/09/21 Пнд 12:52:01 679712510
>>679704
>А окислять его чем?
Его не надо окислять, энергия рекомбинации практически на порядок выше чем при окислении молекулярного водорода. Можно и окислять, конечно, будет небольшой прирост + криогенный кислород можно для охлаждения использовать по идее.
Аноним 13/09/21 Пнд 12:52:33 679713511
Аноним 13/09/21 Пнд 12:59:44 679714512
Аноним 13/09/21 Пнд 13:20:37 679717513
Аноним 13/09/21 Пнд 14:01:28 679723514
>>679700
>но никто не будет его в атмосфере использовать это точно.
Так там не выходит радиоактивное говно наружу. Разве что наведенная радиация, но она спадает быстро.
Аноним 13/09/21 Пнд 14:03:28 679725515
>>679723
Ты про реакторы для нагрева топлива? У них тяги нихуя нет, они годятся для межпланетных.
Аноним 13/09/21 Пнд 17:32:26 679771516
18/09/21 Суб 15:04:17 681076517
Что тут
Аноним 21/09/21 Втр 15:22:07 681684518
>>672710
Кто сказал что пространство "раздувается" с ускорением?
Настройки X
Ответить в тред X
15000
Макс объем: 40Mб, макс кол-во файлов: 4
Кликни/брось файл/ctrl-v
Стикеры X
Избранное / Топ тредов